A principal bundle is a locally trivial family of groups. It turns out that the theory of connections on a vector bundle can be subsumed under the theory of connections on a principal bundle. The latter, moreover, has the advantage that its connection forms are basis-free.

In this chapter we will first give several equivalent constructions of a connection on a principal bundle, and then generalize the notion curvature to a principal bundle, paving the way to a generalization of characteristic classes to principal bundles. Along the way, we also generalize covariant derivatives to principal bundles.

§27 Principal Bundles

We saw in Section 11 that a connection ∇ on a vector bundle E over a manifold M can be represented by a matrix of 1-forms over a framed open set. For any frame e = [e 1 ⋯ e r ] for E over an open set U, the connection matrix ω e relative to e is defined by

$$\displaystyle{\nabla _{X}e_{j} =\sum _{i}(\omega _{e})_{j}^{i}(X)e_{ i}}$$

for all C vector fields X over U. If \(\bar{e} = [\bar{e}_{1}\ \cdots \ \bar{e}_{r}] = ea\) is another frame for E over U, where \(a: U \rightarrow \mathop{\mathrm{GL}}\nolimits (r, \mathbb{R})\) is a matrix of C transition functions, then by Theorem  22.1 the connection matrix ω e transforms according to the rule

$$\displaystyle{\omega _{\bar{e}} = a^{-1}\omega _{ e}a + a^{-1}\,da.}$$

Associated to a vector bundle is an object called its frame bundle π: Fr(E) → M; the total space \(\mathop{\mathrm{Fr}}\nolimits (E)\) of the frame bundle is the set of all ordered bases in the fibers of the vector bundle E → M, with a suitable topology and manifold structure. A section of the frame bundle \(\pi: \mathop{\mathrm{Fr}}\nolimits (E) \rightarrow M\) over an open set U ⊂ M is a map \(s: U \rightarrow \mathop{\mathrm{Fr}}\nolimits (E)\) such that \(\pi \circ s =\mathbb{1}_{U}\), the identity map on U. From this point of view a frame e = [e 1 ⋯ e r ] over U is simply a section of the frame bundle \(\mathop{\mathrm{Fr}}\nolimits (E)\) over U.

Suppose ∇ is a connection on the vector bundle E → M. Miraculously, there exists a matrix-valued 1-form ω on the frame bundle \(\mathop{\mathrm{Fr}}\nolimits (E)\) such that for every frame e over an open set U ⊂ M, the connection matrix ω e of ∇ is the pullback of ω by the section \(e: U \rightarrow \mathop{\mathrm{Fr}}\nolimits (E)\) (Theorem 29.10). This matrix-valued 1-form, called an Ehresmann connection on the frame bundle \(\mathop{\mathrm{Fr}}\nolimits (E)\), is determined uniquely by the connection on the vector bundle E and vice versa. It is an intrinsic object of which a connection matrix ω e is but a local manifestation. The frame bundle of a vector bundle is an example of a principal G-bundle for the group \(G =\mathop{ \mathrm{GL}}\nolimits (r, \mathbb{R})\). The Ehresmann connection on the frame bundle generalizes to a connection on an arbitrary principal bundle.

figure a

Charles Ehresmann (1905–1979)

This section collects together some general facts about principal bundles.

27.1 Principal Bundles

Let E, M, and F be manifolds. We will denote an open cover \(\mathfrak{U}\) of M either as {U α } or more simply as an unindexed set {U} whose general element is denoted by U. A local trivialization with fiber F for a smooth surjection π: E → M is an open cover \(\mathfrak{U} =\{ U\}\) for M together with a collection \(\{\phi _{U}: \pi ^{-1}(U) \rightarrow U \times F\mid U \in \mathfrak{U}\}\) of fiber-preserving diffeomorphisms ϕ U : π −1(U) → U × F:

where η is projection to the first factor. A fiber bundle with fiber F is a smooth surjection π: E → M having a local trivialization with fiber F. We also say that it is locally trivial with fiber F. The manifold E is the total space and the manifold M the base space of the fiber bundle.

The fiber of a fiber bundle π: E → M over x ∈ M is the set E x : = π −1(x). Because π is a submersion, by the regular level set theorem ([21], Th. 9.13, p. 96) each fiber E x is a regular submanifold of E. For x ∈ U, define \(\phi _{U,x}:=\phi _{U}\vert _{E_{x}}: E_{x} \rightarrow \{ x\} \times F\) to be the restriction of the trivialization ϕ U : π −1(U) → U × F to the fiber E x .

Proposition 27.1.

Let π: E → M be a fiber bundle with fiber F. If ϕ U : π −1 (U) → U × F is a trivialization, then ϕ U,x : E x →{ x} × F is a diffeomorphism.

Proof.

The map ϕ U, x is smooth because it is the restriction of the smooth map ϕ U to a regular submanifold. It is bijective because ϕ U is bijective and fiber-preserving. Its inverse ϕ U, x −1 is the restriction of the smooth map ϕ U −1: U × F → π −1(U) to the fiber {x} × F and is therefore also smooth. □ 

A smooth right action of a Lie group G on a manifold M is a smooth map

$$\displaystyle{\mu: M \times G \rightarrow M,}$$

denoted by x ⋅ g: = μ(x, g), such that for all x ∈ M and g, h ∈ G,

(i):

x ⋅ e = x, where e is the identity element of G,

(ii):

(x ⋅ g) ⋅ h = x ⋅ (gh).

We often omit the dot and write more simply xg for x ⋅ g. If there is such a map μ, we also say that G acts smoothly on M on the right. A left action is defined similarly. The stabilizer of a point x ∈ M under an action of G is the subgroup

$$\displaystyle{\mathop{\mathrm{Stab}}\nolimits (x):=\{ g \in G\mid x \cdot g = x\}.}$$

The orbit of x ∈ M is the set

$$\displaystyle{\mathop{\mathrm{Orbit}}\nolimits (x):= xG:=\{ x \cdot g \in M\mid g \in G\}.}$$

Denote by \(\mathop{\mathrm{Stab}}\nolimits (x)\setminus G\) the set of right cosets of \(\mathop{\mathrm{Stab}}\nolimits (x)\) in G. By the orbit-stabilizer theorem, for each x ∈ M the map\(: G \rightarrow \mathop{\mathrm{Orbit}}\nolimits (x)\), gx ⋅ g induces a bijection of sets:

$$\displaystyle\begin{array}{rcl} \mathop{\mathrm{Stab}}\nolimits (x)\setminus G& \longleftrightarrow & \mathop{\mathrm{Orbit}}\nolimits (x), {}\\ \mathop{\mathrm{Stab}}\nolimits (x)g& \longleftrightarrow & x \cdot g. {}\\ \end{array}$$

The action of G on M is free if the stabilizer of every point x ∈ M is the trivial subgroup {e}.

A manifold M together with a right action of a Lie group G on M is called a right G-manifold or simply a G-manifold . A map f: N → M between right G-manifolds is right G-equivariant if

$$\displaystyle{f(x \cdot g) = f(x) \cdot g}$$

for all (x, g) ∈ N × G. Similarly, a map f: N → M between left G-manifolds is left G-equivariant if

$$\displaystyle{f(g \cdot x) = g \cdot f(x)}$$

for all (g, x) ∈ G × N.

A left action can be turned into a right action and vice versa; for example, if G acts on M on the left, then

$$\displaystyle{x \cdot g = g^{-1} \cdot x}$$

is a right action of G on M. Thus, if N is a right G-manifold and M is a left G-manifold, we say a map f: N → M is G-equivariant if

$$\displaystyle{ f(x \cdot g) = f(x) \cdot g = g^{-1} \cdot f(x) }$$
(27.1)

for all (x, g) ∈ N × G.

A smooth fiber bundle π: P → M with fiber G is a smooth principal G-bundle if G acts smoothly and freely on P on the right and the fiber-preserving local trivializations

$$\displaystyle{\phi _{U}: \pi ^{-1}(U) \rightarrow U \times G}$$

are G-equivariant, where G acts on U × G on the right by

$$\displaystyle{(x,h) \cdot g = (x,hg).}$$

Example 27.2 (Product G-bundles).

The simplest example of a principal G-bundle over a manifold M is the product G-bundle η: M × G → M. A trivialization is the identity map on M × G.

Example 27.3 (Homogenous manifolds).

If G is a Lie group and H is a closed subgroup, then the quotient GH can be given the structure of a manifold such that the projection map π: G → GH is a principal H-bundle. This is proven in [22, Th. 3.58, p. 120].

Example 27.4 (Hopf bundle).

The group S 1 of unit complex numbers acts on the complex vector space \(\mathbb{C}^{n+1}\) by left multiplication. This action induces an action of S 1 on the unit sphere S 2n+1 in \(\mathbb{C}^{n+1}\). The complex projective space \(\mathbb{C}P^{n}\) may be defined as the orbit space of S 2n+1 by S 1. The natural projection \(S^{2n+1} \rightarrow \mathbb{C}P^{n}\) with fiber S 1 turn out to be a principal S 1-bundle. When n = 1, \(S^{3} \rightarrow \mathbb{C}P^{1}\) with fiber S 1 is called the Hopf bundle.

Definition 27.5.

Let π Q : Q → N and π P : P → M be principal G-bundles. A morphism of principal G-bundles is a pair of maps \((\bar{f}: Q \rightarrow P,f: N \rightarrow M)\) such that \(\bar{f}: Q \rightarrow P\) is G-equivariant and the diagram

commutes.

Proposition 27.6.

If π: P → M is a principal G-bundle, then the group G acts transitively on each fiber.

Proof.

Since G acts transitively on {x} × G and the fiber diffeomorphism ϕ U, x : P x  → { x} × G is G-equivariant, G must also act transitively on the fiber P x . □ 

Lemma 27.7.

For any group G, a right G-equivariant map f: G → G is necessarily a left translation.

Proof.

Suppose that for all x, g ∈ G,

$$\displaystyle{f(xg) = f(x)g.}$$

Setting x = e, the identity element of G, we obtain

$$\displaystyle{f(g) = f(e)g =\ell _{f(e)}(g),}$$

where f(e): G → G is left translation by f(e). □ 

Suppose {U α } α ∈ A is a local trivialization for a principal G-bundle π: P → M. Whenever the intersection U α β : = U α U β is nonempty, there are two trivializations on π −1(U α β ):

$$\displaystyle{U_{\alpha \beta } \times G\mathop{\longleftarrow }\limits^{\phi _{\alpha }}\pi ^{-1}(U_{\alpha \beta })\mathop{\longrightarrow }\limits^{\phi _{\beta }}U_{\alpha \beta } \times G.}$$

Then ϕ α ϕ β −1: U α β × G → U α β × G is a fiber-preserving right G-equivariant map. By Lemma 27.7, it is a left translation on each fiber. Thus,

$$\displaystyle{ (\phi _{\alpha } \circ \phi _{\beta }^{-1})(x,h) = (x,g_{\alpha \beta }(x)h), }$$
(27.2)

where (x, h) ∈ U α β × G and g α β (x) ∈ G. Because ϕ α ϕ β −1 is a C function of x and h, setting h = e, we see that g α β (x) is a C function of x. The C functions g α β : U α β  → G are called transition functions of the principal bundle π: P → M relative to the trivializing open cover {U α } α ∈ A. They satisfy the cocycle condition: for all α, β, γ ∈ A,

$$\displaystyle{g_{\alpha \beta }g_{\beta \gamma } = g_{\alpha \gamma }\quad \text{ if }U_{\alpha } \cap U_{\beta } \cap U_{\gamma }\neq \varnothing.}$$

From the cocycle condition, one can deduce other properties of the transition functions.

Proposition 27.8.

The transition functions g αβ of a principal bundle π: P → M relative to a trivializing open cover {U α } α∈A satisfy the following properties: for all α,β ∈A,

(i):

g αα = the constant map e,

(ii):

g αβ = g βα −1 if \(U_{\alpha } \cap U_{\beta }\neq \varnothing\).

Proof.

(i):

If α = β = γ, the cocycle condition gives

$$\displaystyle{g_{\alpha \alpha }g_{\alpha \alpha } = g_{\alpha \alpha }.}$$

Hence, g α α  = the constant map e.

(ii):

if γ = α, the cocycle condition gives

$$\displaystyle{g_{\alpha \beta }g_{\beta \alpha } = g_{\alpha \alpha } = e}$$

or

$$\displaystyle{g_{\alpha \beta } = g_{\beta \alpha }^{-1}\quad \text{for }U_{\alpha } \cap U_{\beta }\neq \varnothing.}$$

 □ 

In a principal G-bundle P → M, the group G acts on the right on the total space P, but the transition functions g α β in (27.2) are given by left translations by g α β (x) ∈ G. This phenomenon is a consequence of Lemma 27.7.

27.2 The Frame Bundle of a Vector Bundle

For any real vector space V, let \(\mathop{\mathrm{Fr}}\nolimits (V )\) be the set of all ordered bases in V. Suppose V has dimension r. We will represent an ordered basis v 1, , v r by a row vector v = [v 1 ⋯ v r ], so that the general linear group \(\mathop{\mathrm{GL}}\nolimits (r, \mathbb{R})\) acts on \(\mathop{\mathrm{Fr}}\nolimits (V )\) on the right by matrix multiplication

$$\displaystyle\begin{array}{rcl} v \cdot a& =& [v_{1}\ \cdots \ v_{r}][a_{j}^{i}] {}\\ & =& \Big[\sum v_{i}a_{1}^{i}\ \cdots \ \sum v_{ i}a_{r}^{i}\Big]. {}\\ \end{array}$$

Fix a point \(v \in \mathop{\mathrm{Fr}}\nolimits (V )\). Since the action of \(\mathop{\mathrm{GL}}\nolimits (r, \mathbb{R})\) on \(\mathop{\mathrm{Fr}}\nolimits (V )\) is clearly transitive and free, i.e., \(\mathop{\mathrm{Orbit}}\nolimits (v) =\mathop{ \mathrm{Fr}}\nolimits (V )\) and \(\mathop{\mathrm{Stab}}\nolimits (v) =\{ I\}\), by the orbit-stabilizer theorem there is a bijection

$$\displaystyle\begin{array}{rcl} \phi _{v}: \mathop{\mathrm{GL}}\nolimits (r, \mathbb{R}) = \frac{\mathop{\mathrm{GL}}\nolimits (r, \mathbb{R})} {\mathop{\mathrm{Stab}}\nolimits (v)} & \longleftrightarrow & \mathop{\mathrm{Orbit}}\nolimits (v) =\mathop{ \mathrm{Fr}}\nolimits (V ), {}\\ g& \longleftrightarrow & vg. {}\\ \end{array}$$

Using the bijection ϕ v , we put a manifold structure on \(\mathop{\mathrm{Fr}}\nolimits (V )\) in such a way that ϕ v becomes a diffeomorphism.

If v′ is another element of \(\mathop{\mathrm{Fr}}\nolimits (V )\), then v′ = va for some \(a \in \mathop{\mathrm{GL}}\nolimits (r, \mathbb{R})\) and

$$\displaystyle{\phi _{va}(g) = vag =\phi _{v}(ag) = (\phi _{v} \circ \ell_{a})(g).}$$

Since left multiplication \(\ell_{a}: \mathop{\mathrm{GL}}\nolimits (r, \mathbb{R}) \rightarrow \mathop{\mathrm{GL}}\nolimits (r, \mathbb{R})\) is a diffeomorphism, the manifold structure on \(\mathop{\mathrm{Fr}}\nolimits (V )\) defined by ϕ v is the same as the one defined by ϕ va . We call \(\mathop{\mathrm{Fr}}\nolimits (V )\) with this manifold structure the frame manifold of the vector space V.

Remark 27.9.

A linear isomorphism ϕ: V → W induces a C diffeomorphism \(\widetilde{\phi }:\) \(\mathop{\mathrm{Fr}}\nolimits (V ) \rightarrow \mathop{\mathrm{Fr}}\nolimits (W)\) by

$$\displaystyle{\widetilde{\phi }[v_{1}\ \cdots \ v_{r}] = [\phi (v_{1})\ \cdots \ \phi (v_{r})].}$$

Define an action of \(\mathop{\mathrm{GL}}\nolimits (r, \mathbb{R})\) on \(\mathop{\mathrm{Fr}}\nolimits (\mathbb{R}^{r})\) by

$$\displaystyle{g \cdot [v_{1}\ \cdots \ v_{r}] = [gv_{1}\ \cdots \ gv_{r}].}$$

Thus, if \(\phi: \mathbb{R}^{r} \rightarrow \mathbb{R}^{r}\) is given by left multiplication by \(g \in \mathop{\mathrm{GL}}\nolimits (r, \mathbb{R})\), then so is the induced map \(\widetilde{\phi }\) on the frame manifold \(\mathop{\mathrm{Fr}}\nolimits (\mathbb{R}^{r})\).

Example 27.10 (The frame bundle).

Let η: E → M be a C vector bundle of rank r. We associate to the vector bundle E a C principal \(\mathop{\mathrm{GL}}\nolimits (r, \mathbb{R})\)-bundle \(\pi: \mathop{\mathrm{Fr}}\nolimits (E) \rightarrow M\) as follows. As a set the total space \(\mathop{\mathrm{Fr}}\nolimits (E)\) is defined to be the disjoint union

$$\displaystyle{\mathop{\mathrm{Fr}}\nolimits (E) =\coprod _{x\in M}\mathop{ \mathrm{Fr}}\nolimits (E_{x}).}$$

There is a natural projection map \(\pi: \mathop{\mathrm{Fr}}\nolimits (E) \rightarrow M\) that maps \(\mathop{\mathrm{Fr}}\nolimits (E_{x})\) to {x}.

A local trivialization \(\phi _{\alpha }: E\vert _{U_{\alpha }}\mathop{ \rightarrow }\limits^{\sim } U_{\alpha } \times \mathbb{R}^{r}\) induces a bijection

$$\displaystyle\begin{array}{rcl} & & \qquad \quad \widetilde{\phi _{\alpha }}: \mathop{\mathrm{Fr}}\nolimits (E)\vert _{U_{\alpha }}\mathop{ \rightarrow }\limits^{\sim } U_{\alpha } \times \mathop{\mathrm{Fr}}\nolimits (\mathbb{R}^{r}), {}\\ & & [v_{1}\ \cdots \ v_{r}] \in \mathop{\mathrm{Fr}}\nolimits (E_{x})\mapsto (x,[\phi _{\alpha,x}(v_{1})\ \cdots \ \phi _{\alpha,x}(v_{r})]). {}\\ \end{array}$$

Via \(\widetilde{\phi _{\alpha }}\) one transfers the topology and manifold structure from \(U_{\alpha } \times \mathop{\mathrm{Fr}}\nolimits (\mathbb{R}^{r})\) to \(\mathop{\mathrm{Fr}}\nolimits (E)\vert _{U_{\alpha }}\). This gives \(\mathop{\mathrm{Fr}}\nolimits (E)\) a topology and a manifold structure such that π: Fr(E) → M is locally trivial with fiber \(\mathop{\mathrm{Fr}}\nolimits (\mathbb{R}^{r})\). As the frame manifold \(\mathop{\mathrm{Fr}}\nolimits (\mathbb{R}^{r})\) is diffeomorphic to the general linear group \(\mathop{\mathrm{GL}}\nolimits (r, \mathbb{R})\), it is easy to check that \(\mathop{\mathrm{Fr}}\nolimits (E) \rightarrow M\) is a C principal \(\mathop{\mathrm{GL}}\nolimits (r, \mathbb{R})\)-bundle. We call it the frame bundle of the vector bundle E.

On a nonempty overlap U α β : = U α U β , the transition function for the vector bundle E is the C function \(g_{\alpha \beta }: U_{\alpha \beta } \rightarrow \mathop{\mathrm{GL}}\nolimits (r, \mathbb{R})\) given by

$$\displaystyle\begin{array}{rcl} \phi _{\alpha } \circ \phi _{\beta }^{-1}: U_{\alpha \beta } \times \mathbb{R}^{r}& \rightarrow & U_{\alpha \beta } \times \mathbb{R}^{r}, {}\\ (\phi _{\alpha } \circ \phi _{\beta }^{-1})(x,w)& =& \big(x,g_{\alpha \beta }(x)w\big). {}\\ \end{array}$$

Since the local trivialization for the frame bundle \(\mathop{\mathrm{Fr}}\nolimits (E)\) is induced from the trivialization {U α , ϕ α } for E, the transition functions for \(\mathop{\mathrm{Fr}}\nolimits (E)\) are induced from the transition functions {g α β } for E. By Remark 27.9 the transition functions for the open cover \(\{\mathop{\mathrm{Fr}}\nolimits (E)\vert _{U_{\alpha }}\}\) of \(\mathop{\mathrm{Fr}}\nolimits (E)\) are the same as the transition functions \(g_{\alpha \beta }: U_{\alpha \beta } \rightarrow \mathop{\mathrm{GL}}\nolimits (r, \mathbb{R})\) for the vector bundle E, but now of course \(\mathop{\mathrm{GL}}\nolimits (r, \mathbb{R})\) acts on \(\mathop{\mathrm{Fr}}\nolimits (\mathbb{R}^{r})\) instead of on \(\mathbb{R}^{r}\).

27.3 Fundamental Vector Fields of a Right Action

Suppose G is a Lie group with Lie algebra \(\mathfrak{g}\) and G acts smoothly on a manifold P on the right. To every element \(A \in \mathfrak{g}\) one can associate a vector field A on P called the fundamental vector field on P associated to A: for p in P, define

$$\displaystyle{\underline{A}_{p} = \left. \frac{d} {dt}\right \vert _{t=0}p \cdot e^{tA} \in T_{ p}P.}$$

To understand this equation, first fix a point p ∈ P. Then c p : tp ⋅ e tA is a curve in P with initial point p. By definition, the vector A p is the initial vector of this curve. Thus,

$$\displaystyle{\underline{A}_{p} = c_{p}^{\prime}(0) = c_{p{\ast}}\left (\left. \frac{d} {dt}\right \vert _{t=0}\right ) \in T_{p}P.}$$

As a tangent vector at p is a derivation on germs of C functions at p, in terms of a C function f at p,

$$\displaystyle{\underline{A}_{p}f = c_{p{\ast}}\left (\left. \frac{d} {dt}\right \vert _{t=0}\right )f = \left. \frac{d} {dt}\right \vert _{t=0}f \circ c_{p} = \left. \frac{d} {dt}\right \vert _{t=0}f(p \cdot e^{tA}).}$$

Proposition 27.11.

For each \(A \in \mathfrak{g}\) , the fundamental vector field A is C on P.

Proof.

It suffices to show that for every C function f on P, the function A f is also C on P. Let μ: P × G → P be the C map defining the right action of G on P. For any p in P,

$$\displaystyle{\underline{A}_{p}f = \left. \frac{d} {dt}\right \vert _{t=0}f(p \cdot e^{tA}) = \left. \frac{d} {dt}\right \vert _{t=0}(f \circ \mu )(p,e^{tA}).}$$

Since e tA is a C function of t, and f and μ are both C , the derivative

$$\displaystyle{ \frac{\ d} {dt}(f \circ \mu )(p,e^{tA})}$$

is C in p and in t. Therefore, A p f is a C function of p. □ 

Recall that \(\mathfrak{X}(P)\) denotes the Lie algebra of C vector fields on the manifold P. The fundamental vector field construction gives rise to a map

$$\displaystyle{\sigma: \mathfrak{g} \rightarrow \mathfrak{X}(P),\quad \sigma (A):=\underline{ A}.}$$

For p in P, define j p : G → P by j p (g) = p ⋅ g. Computing the differential j p using the curve c(t) = e tA, we obtain the expression

$$\displaystyle{ j_{p{\ast}}(A) = \left. \frac{d} {dt}\right \vert _{t=0}j_{p}(e^{tA}) = \left. \frac{d} {dt}\right \vert _{t=0}p \cdot e^{tA} =\underline{ A}_{ p}. }$$
(27.3)

This alternate description of fundamental vector fields, A p  = j p(A), shows that the map \(\sigma: \mathfrak{g} \rightarrow \mathfrak{X}(P)\) is linear over \(\mathbb{R}\). In fact, σ is a Lie algebra homomorphism (Problem 27.1).

Example 27.12.

Consider the action of a Lie group G on itself by right multiplication. For p ∈ G, the map j p : G → G, j p (g) = p ⋅ g =  p (g) is simply left multiplication by p. By (27.3), for \(A \in \mathfrak{g}\), A p  =  p(A). Thus, for the action of G on G by right multiplication, the fundamental vector field A on G is precisely the left-invariant vector field generated by A. In this sense the fundamental vector field of a right action is a generalization of a left-invariant vector field on a Lie group.

For g in a Lie group G, let c g : G → G be conjugation by g: c g (x) = gxg −1. The adjoint representation is defined to be the differential of the conjugation map: \(\mathop{\mathrm{Ad}}\nolimits (g) = (c_{g})_{{\ast}}: \mathfrak{g} \rightarrow \mathfrak{g}\).

Proposition 27.13.

Suppose a Lie group G acts smoothly on a manifold P on the right. Let r g : P → P be the right translation r g (p) = p ⋅ g. For \(A \in \mathfrak{g}\) the fundamental vector field A on P satisfies the following equivariance property:

$$\displaystyle{r_{g{\ast}}\underline{A} =\underline{ (\mathop{\mathrm{Ad}}\nolimits g^{-1})A}.}$$

Proof.

We need to show that for every p in P, \(r_{g{\ast}}(\underline{A}_{p}) =\underline{ (\mathop{\mathrm{Ad}}\nolimits g^{-1})A}_{pg}\). For x in G,

$$\displaystyle{(r_{g} \circ j_{p})(x) = pxg = pgg^{-1}xg = j_{ pg}(g^{-1}xg) = (j_{ pg} \circ c_{g^{-1}})(x).}$$

By the chain rule,

$$\displaystyle{r_{g{\ast}}(\underline{A}_{p}) = r_{g{\ast}}j_{p{\ast}}(A) = j_{pg{\ast}}(c_{g^{-1}})_{{\ast}}(A) = j_{pg{\ast}}\big((\mathop{\mathrm{Ad}}\nolimits g^{-1})A\big) =\underline{ (\mathop{\mathrm{Ad}}\nolimits g^{-1})A}_{ pg}.}$$

 □ 

27.4 Integral Curves of a Fundamental Vector Field

In this section suppose a Lie group G with Lie algebra \(\mathfrak{g}:=\mathrm{ Lie}(G)\) acts smoothly on the right on a manifold P.

Proposition 27.14.

For p ∈ P and \(A \in \mathfrak{g}\) , the curve c p (t) = p ⋅ e tA, \(t \in \mathbb{R}\) , is the integral curve of the fundamental vector field A through p.

Proof.

We need to show that \(c_{p}^{\prime}(t) =\underline{ A}_{c_{p}(t)}\) for all \(t \in \mathbb{R}\) and all p ∈ P. It is essentially a sequence of definitions:

$$\displaystyle{c_{p}^{\prime}(t) = \left. \frac{d} {ds}\right \vert _{s=0}c_{p}(t + s) = \left. \frac{d} {ds}\right \vert _{s=0}pe^{tA}e^{sA} =\underline{ A}_{ pe^{tA}} =\underline{ A}_{c_{p}(t)}.}$$

 □ 

Proposition 27.15.

The fundamental vector field A on a manifold P vanishes at a point p in P if and only if A is in the Lie algebra of \(\mathop{\mathrm{Stab}}\nolimits (p)\).

Proof.

( ⇐ ) If \(A \in \mathrm{Lie}\big(\mathop{\mathrm{Stab}}\nolimits (p)\big)\), then \(e^{tA} \in \mathop{\mathrm{Stab}}\nolimits (p)\), so

$$\displaystyle{\underline{A}_{p} = \left. \frac{d} {dt}\right \vert _{t=0}p \cdot e^{tA} = \left. \frac{d} {dt}\right \vert _{t=0}p = 0.}$$

( ⇒ ) Suppose A p  = 0. Then the constant map γ(t) = p is an integral curve of A through p, since

$$\displaystyle{\gamma ^{\prime}(t) = 0 =\underline{ A}_{p} =\underline{ A}_{\gamma (t)}.}$$

On the other hand, by Proposition 27.14, c p (t) = p ⋅ e tA is also an integral curve of A through p. By the uniqueness of the integral curve through a point, c p (t) = γ(t) or p ⋅ e tA = p for all \(t \in \mathbb{R}\). This implies that \(e^{tA} \in \mathop{\mathrm{Stab}}\nolimits (p)\) and therefore \(A \in \mathrm{Lie}\big(\mathop{\mathrm{Stab}}\nolimits (p)\big)\). □ 

Corollary 27.16.

For a right action of a Lie group G on a manifold P, let p ∈ P and j p : G → P be the map j p (g) = p ⋅ g. Then the kernel ker j p∗ of the differential of j p at the identity

$$\displaystyle{j_{p{\ast}} = (j_{p})_{{\ast},e}: \mathfrak{g} \rightarrow T_{p}P}$$

is \(\mathrm{Lie}\big(\mathop{\mathrm{Stab}}\nolimits (p)\big)\).

Proof.

For \(A \in \mathfrak{g}\), we have A p  = j p(A) by (27.3). Thus,

$$\displaystyle\begin{array}{rcl} A \in \ker j_{p{\ast}}\quad & \Longleftrightarrow& \quad j_{p{\ast}}(A) = 0 {}\\ \quad \quad & \Longleftrightarrow& \quad \underline{A}_{p} = 0 {}\\ \quad & \Longleftrightarrow& \quad A \in \mathrm{Lie}\big(\mathop{\mathrm{Stab}}\nolimits (p)\big)\quad \mathrm{(by Proposition\ 27.15)}. {}\\ \end{array}$$

 □ 

27.5 Vertical Subbundle of the Tangent Bundle TP

Throughout this section, G is a Lie group with Lie algebra \(\mathfrak{g}\) and π: P → M is a principal G-bundle. On the total space P there is a natural notion of vertical tangent vectors. We will show that the vertical tangent vectors on P form a trivial subbundle of the tangent bundle TP.

By the local triviality of a principal bundle, at every point p ∈ P the differential π ∗, p : T p P → T π(p) M of the projection π is surjective. The vertical tangent subspace \(\mathcal{V}_{p} \subset T_{p}P\) is defined to be kerπ ∗, p . Hence, there is a short exact sequence of vector spaces

$$\displaystyle{ 0 \rightarrow \mathcal{V}_{p}\longrightarrow T_{p}P\mathop{\longrightarrow}\limits_{}^{\pi _{{\ast},p}}T_{\pi (p)}M \rightarrow 0, }$$
(27.4)

and

$$\displaystyle{\dim \mathcal{V}_{p} =\dim T_{p}P -\dim T_{\pi (p)}M =\dim G.}$$

An element of \(\mathcal{V}_{p}\) is called a vertical tangent vector at p.

Proposition 27.17.

For any \(A \in \mathfrak{g}\) , the fundamental vector field A is vertical at every point p ∈ P.

Proof.

With j p : G → P defined as usual by j p (g) = p ⋅ g,

$$\displaystyle{(\pi \circ j_{p})(g) =\pi (p \cdot g) =\pi (p).}$$

Since A p  = j p(A) by (27.3), and πj p is a constant map,

$$\displaystyle{\pi _{{\ast},p}(\underline{A}_{p}) = (\pi _{{\ast},p} \circ j_{p{\ast}})(A) = (\pi \circ j_{p})_{{\ast}}(A) = 0.}$$

 □ 

Thus, in case P is a principal G-bundle, we can refine Corollary 27.16 to show that j p maps \(\mathfrak{g}\) into the vertical tangent space:

$$\displaystyle{(j_{p})_{{\ast},e}: \mathfrak{g} \rightarrow \mathcal{V}_{p} \subset T_{p}P.}$$

In fact, this is an isomorphism.

Proposition 27.18.

For p ∈ P, the differential at e of the map j p : G → P is an isomorphism of \(\mathfrak{g}\) onto the vertical tangent space: \(j_{p{\ast}} = (j_{p})_{{\ast},e}: \mathfrak{g}\mathop{ \rightarrow }\limits^{\sim }\mathcal{V}_{p}\).

Proof.

By Corollary 27.16, \(\ker j_{p{\ast}} = \mathrm{Lie}\big(\mathop{\mathrm{Stab}}\nolimits (p)\big)\). Since G acts freely on P, the stabilizer of any point p ∈ P is the trivial subgroup {e}. Thus, kerj p = 0 and j p is injective. By Proposition 27.17, the image j p lies in the vertical tangent space \(\mathcal{V}_{p}\). Since \(\mathfrak{g}\) and \(\mathcal{V}_{p}\) have the same dimension, the injective linear map \(j_{p{\ast}}: \mathfrak{g} \rightarrow \mathcal{V}_{p}\) has to be an isomorphism. □ 

Corollary 27.19.

The vertical tangent vectors at a point of a principal bundle are precisely the fundamental vectors.

Let B 1, , B be a basis for the Lie algebra \(\mathfrak{g}\). By the proposition, the fundamental vector fields B 1, , B on P form a basis of \(\mathcal{V}_{p}\) at every point p ∈ P. Hence, they span a trivial subbundle \(\mathcal{V}:=\coprod _{p\in P}\mathcal{V}_{p}\) of the tangent bundle TP. We call \(\mathcal{V}\) the vertical subbundle of TP.

As we learned in Section 20.5, the differential π : TP → TM of a C map π: P → M induces a bundle map \(\tilde{\pi }_{{\ast}}: TP \rightarrow \pi ^{{\ast}}TM\) over P, given by

The map \(\tilde{\pi }_{{\ast}}\) is surjective because it sends the fiber T p P onto the fiber (π TM) p  ≃ T π(p) M. Its kernel is precisely the vertical subbundle \(\mathcal{V}\) by (27.4). Hence, \(\mathcal{V}\) fits into a short exact sequence of vector bundles over P:

$$\displaystyle{ 0 \rightarrow \mathcal{V}\longrightarrow TP\mathop{\longrightarrow }\limits^{\tilde{\pi }_{{\ast}}}\pi ^{{\ast}}TM \rightarrow 0. }$$
(27.5)

27.6 Horizontal Distributions on a Principal Bundle

On the total space P of a smooth principal bundle π: P → M, there is a well-defined vertical subbundle \(\mathcal{V}\) of the tangent bundle TP. We call a subbundle \(\mathcal{H}\) of TP a horizontal distribution on P if \(TP = \mathcal{V}\oplus \mathcal{H}\) as vector bundles; in other words, \(T_{p}P = \mathcal{V}_{p} + \mathcal{H}_{p}\) and \(\mathcal{V}_{p} \cap \mathcal{H}_{p} = 0\) for every p ∈ P. In general, there is no canonically defined horizontal distribution on a principal bundle.

A splitting of a short exact sequence of vector bundles \(0 \rightarrow A\mathop{ \rightarrow }\limits^{ i}B\mathop{ \rightarrow }\limits^{ j}C \rightarrow 0\) over a manifold P is a bundle map k: C → B such that \(j \circ k =\mathbb{1}_{C}\), the identity bundle map on C.

Proposition 27.20.

Let

$$\displaystyle{ 0 \rightarrow A\mathop{ \rightarrow }\limits^{ i}B\mathop{ \rightarrow }\limits^{ j}C \rightarrow 0 }$$
(27.6)

be a short exact sequence of vector bundles over a manifold P. Then there is a one-to-one correspondence

$$\displaystyle{\{\mbox{ subbundles }H \subset B\mid B = i(A) \oplus H\}\longleftrightarrow \{\mbox{ splittings $k: C \rightarrow B$ of (27.6)}\}.}$$

Proof.

If H is a subbundle of B such that B = i(A) ⊕ H, then there are bundle isomorphisms H ≃ Bi(A) ≃ C. Hence, C maps isomorphically onto H in B. This gives a splitting k: C → B.

If k: C → B is a splitting, let H: = k(C), which is a subbundle of B. Moreover, if i(a) = k(c) for some a ∈ A and c ∈ C, then

$$\displaystyle{0 = ji(a) = jk(c) = c.}$$

Hence, i(A) ∩ k(C) = 0.

Finally, to show that B = i(A) + k(C), let b ∈ B. Then

$$\displaystyle{j\big(b - kj(b)\big) = j(b) - j(b) = 0.}$$

By the exactness of (27.6), bkj(b) = i(a) for some a ∈ A. Thus,

$$\displaystyle{b = i(a) + kj(b) \in i(A) + k(C).}$$

This proves that B = i(A) + k(C) and therefore B = i(A) ⊕ k(C). □ 

As we just saw in the preceding section, for every principal bundle π: P → M the vertical subbundle \(\mathcal{V}\) fits into a short exact sequence (27.5) of vector bundles over P. By Proposition 27.20, there is a one-to-one correspondence between horizontal distributions on P and splittings of the sequence (27.5).

Problems

27.1. Lie bracket of fundamental vector fields

Let G be a Lie group with Lie algebra \(\mathfrak{g}\) and let P be a manifold on which G acts on the right. Prove that for \(A,B \in \mathfrak{g}\),

$$\displaystyle{\underline{[A,B]} = [\underline{A},\underline{B}].}$$

Hence, the map \(\sigma: \mathfrak{g} \rightarrow \mathfrak{X}(P)\), AA is a Lie algebra homomorphism.

27.2. Short exact sequence of vector spaces

Prove that if \(0 \rightarrow A\mathop{ \rightarrow }\limits^{ i}B\mathop{ \rightarrow }\limits^{ j}C \rightarrow 0\) is a short exact sequence of finite-dimensional vector spaces, then dimB = dimA + dimC.

27.3. Splitting of a short exact sequence

Suppose \(0 \rightarrow A\mathop{ \rightarrow }\limits^{ i}B \rightarrow C \rightarrow 0\) is a short exact sequence of vector bundles over a manifold P. A retraction of i: A → B is a map \(r: B\mathop{ \rightarrow }\limits^{ j}A\) such that \(r \circ i =\mathbb{1}_{A}\). Show that i has a retraction if and only if the sequence has a splitting.

27.4. The differential of an action

Let μ: P × G → P be an action of a Lie group G on a manifold P. For g ∈ G, the tangent space T g G may be identified with \(\ell_{g{\ast}}\mathfrak{g}\), where g : G → G is left multiplication by g ∈ G and \(\mathfrak{g} = T_{e}G\) is the Lie algebra of G. Hence, an element of the tangent space T (p, g)(P × G) is of the form (X p ,  g A) for X p  ∈ T p P and \(A \in \mathfrak{g}\). Prove that the differential

$$\displaystyle{\mu _{{\ast}} =\mu _{{\ast},(p,g)}: T_{(p,g)}(P \times G) \rightarrow T_{pg}P}$$

is given by

$$\displaystyle{\mu _{{\ast}}(X_{p},\ell_{g{\ast}}A) = r_{g{\ast}}(X_{p}) +\underline{ A}_{pg}.}$$

27.5. Fundamental vector field under a trivialization

Let ϕ α : π −1 U α  → U α × G

$$\displaystyle{\phi _{\alpha }(p) =\big (\pi (p),g_{\alpha }(p)\big)}$$

be a trivialization of π −1 U ga in a principal bundle P. Let \(A \in \mathfrak{g}\), the Lie algebra of G and A the fundamental vector field on P that it induces. Prove that

$$\displaystyle{g_{\alpha {\ast}}(\underline{A}_{p}) =\ell _{g_{\alpha }(p){\ast}}(A) \in T_{g_{\alpha }(p)}(G).}$$

27.6. Trivial principal bundle

Prove that a principal bundle π: P → M is trivial if and only if it has a section.

27.7. Pullback of a principal bundle to itself

Prove that if π: P → M is a principal bundle, then the pullback bundle π P → P is trivial.

27.8. Quotient space of a principal bundle

Let G be a Lie group and H a closed subgroup. Prove that if π P → M is a principal G-bundle, then P → PH is a principal H-subbundle.

27.9. Fundamental vector fields

Let N and M be G-manifolds with G acting on the right. If \(A \in \mathfrak{g}\) and f: N → M is G-equivariant, then

$$\displaystyle{f_{{\ast}}(\underline{A}_{N,q}) =\underline{ A}_{M,f(q)}.}$$

§28 Connections on a Principal Bundle

Let G be a Lie group with Lie algebra \(\mathfrak{g}\). As we saw in the preceding section, on a principal G-bundle P → M, the notion of a vertical tangent vector is well defined, but not that of a horizontal tangent vector. A connection on a principal bundle is essentially the choice of a horizontal complement to the vertical tangent bundle on P. Alternatively, it can be given by a \(\mathfrak{g}\)-valued 1-form on P. In this section we will study these two equivalent manifestations of a connection:

(i):

a smooth right-invariant horizontal distribution on P,

(ii):

a smooth G-equivariant \(\mathfrak{g}\)-valued 1-form ω on P such that on the fundamental vector fields,

$$\displaystyle{ \omega (\underline{A}) = A\quad \text{for all }A \in \mathfrak{g}. }$$
(28.1)

Under the identification of \(\mathfrak{g}\) with a vertical tangent space, condition (28.1) says that ω restricts to the identity map on vertical vectors.

The correspondence between (i) and (ii) is easy to describe. Given a right-invariant horizontal distribution \(\mathcal{H}\) on P, we define a \(\mathfrak{g}\)-valued 1-form ω on P to be, at each point p, the projection with kernel \(\mathcal{H}_{p}\) from the tangent space to the vertical space. Conversely, given a right-equivariant \(\mathfrak{g}\)-valued 1-form ω that is the identity on the vertical space at each point p ∈ P, we define a horizontal distribution \(\mathcal{H}\) on P to be kerω p at each p ∈ P.

28.1 Connections on a Principal Bundle

Let G be a Lie group with Lie algebra \(\mathfrak{g}\), and let π: P → M be a principal G-bundle. A distribution on a manifold is a subbundle of the tangent bundle. Recall that a distribution \(\mathcal{H}\) on P is horizontal if it is complementary to the vertical subbundle \(\mathcal{V}\) of the tangent bundle TP: for all p in P,

$$\displaystyle{T_{p}P = \mathcal{V}_{p} \oplus \mathcal{H}_{p}.}$$

Suppose \(\mathcal{H}\) is a horizontal distribution on the total space P of a principal G-bundle π: P → M. For p ∈ P, if j p : G → P is the map j p (g) = p ⋅ g, then the vertical tangent space \(\mathcal{V}_{p}\) can be canonically identified with the Lie algebra \(\mathfrak{g}\) via the isomorphism \(j_{p{\ast}}: \mathfrak{g} \rightarrow \mathcal{V}_{p}\) (Proposition 27.18). Let \(v: T_{p}P = \mathcal{V}_{p} \oplus \mathcal{H}_{p} \rightarrow \mathcal{V}_{p}\) be the projection to the vertical tangent space with kernel \(\mathcal{H}_{p}\). For Y p  ∈ T p P, v(Y p ) is called the vertical component of Y p . (Although the vertical subspace \(\mathcal{V}_{p}\) is intrinsically defined, the notion of the vertical component of a tangent vector depends on the choice of a horizontal complement \(\mathcal{H}_{p}\).) If ω p is the composite

$$\displaystyle{ \omega _{p}:= j_{p{\ast}}^{-1} \circ v: T_{ p}P\mathop{ \rightarrow }\limits^{ v}\mathcal{V}_{p}\mathop{ \rightarrow }\limits^{ j_{p{\ast}}^{-1}}\mathfrak{g}, }$$
(28.2)

then ω is a \(\mathfrak{g}\)-valued 1-form on P. In terms of ω, the vertical component of Y p  ∈ T p P is

$$\displaystyle{ v(Y _{p}) = j_{p{\ast}}\big(\omega _{p}(Y _{p})\big). }$$
(28.3)

Theorem 28.1.

If \(\mathcal{H}\) is a smooth right-invariant horizontal distribution on the total space P of a principal G-bundle π: P → M, then the \(\mathfrak{g}\) -valued 1-form ω on P defined above satisfies the following three properties:

(i):

for any \(A \in \mathfrak{g}\) and p ∈ P, we have ω p(A p) = A;

(ii):

(G-equivariance) for any g ∈ G, \(r_{g}^{{\ast}}\omega = (\mathop{\mathrm{Ad}}\nolimits g^{-1})\omega\);

(iii):

ω is C .

Proof.

(i) Since A p is already vertical (Proposition 27.17), the projection v leaves it invariant, so

$$\displaystyle{\omega _{p}(\underline{A}_{p}) = j_{p{\ast}}^{-1}\big(v(\underline{A}_{ p})\big) = j_{p{\ast}}^{-1}(\underline{A}_{ p}) = A.}$$

(ii) For p ∈ P and Y p  ∈ T p P, we need to show

$$\displaystyle{\omega _{pg}(r_{g{\ast}}Y _{p}) = (\mathop{\mathrm{Ad}}\nolimits g^{-1})\omega _{ p}(Y _{p}).}$$

Since both sides are \(\mathbb{R}\)-linear in Y p and Y p is the sum of a vertical and a horizontal vector, we may treat these two cases separately.

If Y p is vertical, then by Proposition 27.18, Y p  = A p for some \(A \in \mathfrak{g}\). In this case

$$\displaystyle\begin{array}{rcl} \omega _{pg}(r_{g{\ast}}\underline{A}_{p})& =& \omega _{pg}\left (\underline{(\mathop{\mathrm{Ad}}\nolimits g^{-1})A}_{ pg}\right )\quad\ \mathrm{(by Proposition\ 27.13)} {}\\ & =& (\mathop{\mathrm{Ad}}\nolimits g^{-1})A\qquad \qquad \quad \text{(by (i))} {}\\ & =& (\mathop{\mathrm{Ad}}\nolimits g^{-1})\omega _{ p}(\underline{A}_{p})\qquad \ \ \text{(by (i) again).} {}\\ \end{array}$$

If Y p is horizontal, then by the right-invariance of the horizontal distribution \(\mathcal{H}\), so is r g Y p . Hence,

$$\displaystyle{\omega _{pg}(r_{g{\ast}}Y _{p}) = 0 = (\mathop{\mathrm{Ad}}\nolimits g^{-1})\omega _{ p}(Y _{p}).}$$

(iii) Fix a point p ∈ P. We will show that ω is C in a neighborhood of p. Let B 1, , B be a basis for the Lie algebra \(\mathfrak{g}\) and B 1, , B the associated fundamental vector fields on P. By Proposition 27.11, these vector fields are all C on P. Since \(\mathcal{H}\) is a C distribution on P, one can find a neighborhood W of p and C horizontal vector fields X 1, , X n on W that span \(\mathcal{H}\) at every point of W. Then B 1, , B , X 1, , X n is a C frame for the tangent bundle TP over W. Thus, any C vector field X on W can be written as a linear combination

$$\displaystyle{X =\sum a^{i}\underline{B_{ i}} +\sum b^{j}X_{ j}}$$

with C coefficients a i, b j on W. By the definition of ω,

$$\displaystyle{\omega (X) =\omega \left (\sum a^{i}\underline{B_{ i}}\right ) =\sum a^{i}B_{ i}.}$$

This proves that ω is a C 1-form on W. □ 

Note that in this theorem the proof of the smoothness of ω requires only that the horizontal distribution \(\mathcal{H}\) be smooth; it does not use the right-invariance of \(\mathcal{H}\).

Definition 28.2.

An Ehresmann connection or simply a connection on a principal G-bundle P → M is a \(\mathfrak{g}\)-valued 1-form ω on P satisfying the three properties of Theorem 28.1.

A \(\mathfrak{g}\)-valued 1-form α on P can be viewed as a map \(\alpha: TP \rightarrow \mathfrak{g}\) from the tangent bundle TP to the Lie algebra \(\mathfrak{g}\). Now both TP and \(\mathfrak{g}\) are G-manifolds: the Lie group G acts on TP on the right by the differentials of right translations and it acts on \(\mathfrak{g}\) on the left by the adjoint representation. By (27.1), \(\alpha: TP \rightarrow \mathfrak{g}\) is G-equivariant if and only if for all p ∈ P, X p  ∈ T p P, and g ∈ G,

$$\displaystyle{\alpha (X_{p} \cdot g) = g^{-1} \cdot \alpha (X_{ p}),}$$

or

$$\displaystyle{\alpha (r_{g{\ast}}X_{p}) = (\mathop{\mathrm{Ad}}\nolimits g^{-1})\alpha (X_{ p}).}$$

Thus, \(\alpha: TP \rightarrow \mathfrak{g}\) is G-equivariant if and only if \(r_{g}^{{\ast}}\alpha = (\mathop{\mathrm{Ad}}\nolimits g^{-1})\alpha\) for all g ∈ G. Condition (ii) of a connection ω on a principal bundle says precisely that ω is G-equivariant as a map from TP to \(\mathfrak{g}\).

28.2 Vertical and Horizontal Components of a Tangent Vector

As we noted in Section 27.5, on any principal G-bundle π: P → M, the vertical subspace \(\mathcal{V}_{p}\) of the tangent space T p P is intrinsically defined:

$$\displaystyle{\mathcal{V}_{p}:=\ker \pi _{{\ast}}: T_{p}P \rightarrow T_{\pi (p)}M.}$$

By Proposition 27.18, the map j p naturally identifies the Lie algebra \(\mathfrak{g}\) of G with the vertical subspace \(\mathcal{V}_{p}\).

In the presence of a horizontal distribution on the total space P of a principal bundle, every tangent vector Y p  ∈ T p P decomposes uniquely into the sum of a vertical vector and a horizontal vector:

$$\displaystyle{Y _{p} = v(Y _{p}) + h(Y _{p}) \in \mathcal{V}_{p} \oplus \mathcal{H}_{p}.}$$

These are called, respectively, the vertical component and horizontal component of the vector Y p . As p varies over P, this decomposition extends to a decomposition of a vector field Y on P:

$$\displaystyle{Y = v(Y ) + h(Y ).}$$

We often omit the parentheses in v(Y ) and h(Y ), and write vY and hY instead.

Proposition 28.3.

If \(\mathcal{H}\) is a C horizontal distribution on the total space P of a principal bundle, then the vertical and horizontal components v(Y ) and h(Y ) of a C vector field Y on P are also C .

Proof.

Let ω be the \(\mathfrak{g}\)-valued 1-form associated to the horizontal distribution \(\mathcal{H}\) by (28.2). It is C by Theorem 28.1(iii). In terms of a basis B 1, , B for \(\mathfrak{g}\), we can write ω = ∑ ω i B i , where ω i are C 1-forms on P. If Y p  ∈ T p P, then by (28.3) its vertical component v(Y p ) is

$$\displaystyle{v(Y _{p}) = j_{p{\ast}}(\omega _{p}(Y _{p})) = j_{p{\ast}}\left (\sum \omega _{p}^{i}(Y _{ p})B_{i}\right ) =\sum \omega _{ p}^{i}(Y _{ p})(\underline{B_{i}})_{p}.}$$

As p varies over P,

$$\displaystyle{v(Y ) =\sum \omega ^{i}(Y )\underline{B_{ i}}.}$$

Since ω i, Y, and B i are all C , so is v(Y ). Because h(Y ) = Yv(Y ), the horizontal component h(Y ) of a C vector field Y on P is also C . □ 

On a principal bundle π: P → M, if r g : P → P is right translation by g ∈ G, then πr g  = π. It follows that π r g = π . Thus, the right translation r g: T p P → T pg P sends a vertical vector to a vertical vector. By hypothesis, \(r_{g{\ast}}\mathcal{H}_{p} = \mathcal{H}_{pg}\) and hence the right translation r g also sends a horizontal vector to a horizontal vector.

Proposition 28.4.

Suppose \(\mathcal{H}\) is a smooth right-invariant horizontal distribution on the total space of a principal G-bundle π: P → M. For each g ∈ G, the right translation r g∗ commutes with the projections v and h.

Proof.

Any X p  ∈ T p P decomposes into vertical and horizontal components:

$$\displaystyle{X_{p} = v(X_{p}) + h(X_{p}).}$$

Applying r g to both sides, we get

$$\displaystyle{ r_{g{\ast}}X_{p} = r_{g{\ast}}v(X_{p}) + r_{g{\ast}}h(X_{p}). }$$
(28.4)

Since r g preserves vertical and horizontal subspaces, r g v(X p ) is vertical and r g h(X p ) is horizontal. Thus, (28.4) is the decomposition of r g X p into vertical and horizontal components. This means for every X p  ∈ T p P,

$$\displaystyle{vr_{g{\ast}}(X_{p}) = r_{g{\ast}}v(X_{p})\quad \text{and}\quad hr_{g{\ast}}(X_{p}) = r_{g{\ast}}h(X_{p}).}$$

 □ 

28.3 The Horizontal Distribution of an Ehresmann Connection

In Section 28.1 we showed that a smooth, right-invariant horizontal distribution on the total space of a principal bundle determines an Ehresmann connection. We now prove the converse.

Theorem 28.5.

If ω is a connection on the principal G-bundle π: P → M, then \(\mathcal{H}_{p}:=\ker \omega _{p}\) , p ∈ P, is a smooth right-invariant horizontal distribution on P.

Proof.

We need to verify three properties:

(i):

At each point p in P, the tangent space T p P decomposes into a direct sum \(T_{p}P = \mathcal{V}_{p} \oplus \mathcal{H}_{p}\).

(ii):

For p ∈ P and g ∈ G, \(r_{g{\ast}}(\mathcal{H}_{p}) \subset \mathcal{H}_{pg}\).

(iii):

\(\mathcal{H}\) is a C subbundle of the tangent bundle TP.

(i) Since \(\mathcal{H}_{p} =\ker \omega _{p}\), there is an exact sequence

$$\displaystyle{0 \rightarrow \mathcal{H}_{p} \rightarrow T_{p}P\mathop{ \rightarrow }\limits^{\omega _{p}}\mathfrak{g} \rightarrow 0.}$$

The map \(j_{p{\ast}}: \mathfrak{g} \rightarrow \mathcal{V}_{p} \subset T_{p}P\) provides a splitting of the sequence. By Proposition 27.20, there is a sequence of isomorphisms

$$\displaystyle{T_{p}P \simeq \mathfrak{g} \oplus \mathcal{H}_{p} \simeq \mathcal{V}_{p} \oplus \mathcal{H}_{p}.}$$

(ii) Suppose \(Y _{p} \in \mathcal{H}_{p} =\ker \omega _{p}\). By the right-equivariance property of an Ehresmann connection,

$$\displaystyle{\omega _{pg}(r_{g{\ast}}Y _{p}) = (r_{g}^{{\ast}}\omega )_{ p}(Y _{p}) = (\mathop{\mathrm{Ad}}\nolimits g^{-1})\omega _{ p}(Y _{p}) = 0.}$$

Hence, \(r_{g{\ast}}Y _{p} \in \mathcal{H}_{pg}\).

(iii) Let B 1, , B be a basis for the Lie algebra \(\mathfrak{g}\) of G. Then ω = ∑ ω i B i , where ω 1, , ω are smooth \(\mathbb{R}\)-valued 1-forms on P and for p ∈ P,

$$\displaystyle{\mathcal{H}_{p} =\bigcap _{ i=1}^{\ell}\ker \omega _{ p}^{i}.}$$

Since \(\omega _{p}: T_{p}P \rightarrow \mathfrak{g}\) is surjective, ω 1, , ω are linearly independent at p.

Fix a point p ∈ P and let x 1, , x m be local coordinates near p on P. Then

$$\displaystyle{\omega ^{i} =\sum _{ j=1}^{m}f_{ j}^{i}\,dx^{j},\quad i = 1,\ldots,\ell}$$

for some C functions f j i in a neighborhood of p.

Let b 1, , b m be the fiber coordinates of TP near p, i.e., if v q  ∈ T q P for q near p, then

$$\displaystyle{v_{q} =\sum b^{j}\left. \frac{\partial \ } {\partial x^{j}}\right \vert _{q}.}$$

In terms of local coordinates,

$$\displaystyle\begin{array}{rcl} \mathcal{H}_{q} =\bigcap _{ i=1}^{\ell}\ker \omega _{ q}^{i}& =& \{v_{ q} \in T_{q}P\mid \omega _{q}^{i}(v_{ q}) = 0,i = 1,\ldots,\ell\} {}\\ & =& \{(b^{1},\ldots,b^{m}) \in \mathbb{R}^{m}\mid \sum _{ j=1}^{m}f_{ j}^{i}(q)b^{j} = 0,i = 1,\ldots,\ell\}. {}\\ \end{array}$$

Let F i(q, b) =  j = 1 m f j i(q)b j, i = 1, , . Since ω 1, , ω are linearly independent at p, the Jacobian matrix [∂ F i∂ b j] = [f j i], an × m matrix, has rank at p. Without loss of generality, we may assume that the first × block of [f j i(p)] has rank . Since having maximal rank is an open condition, there is a neighborhood U p of p on which the first × block of [f j i] has rank . By the implicit function theorem, on U p , b 1, , b are C functions of b +1, , b m, say

$$\displaystyle\begin{array}{rcl} & & b^{1} = b^{1}(b^{\ell+1},\ldots,b^{m}), {}\\ & & \quad \vdots {}\\ & & b^{\ell} = b^{\ell}(b^{\ell+1},\ldots,b^{m}). {}\\ \end{array}$$

Let

$$\displaystyle\begin{array}{rcl} X_{1}& =& \sum _{j=1}^{\ell}b^{j}(1,0,\ldots,0) \frac{\partial \ } {\partial x^{j}} + \frac{\partial \ } {\partial x^{\ell+1}} {}\\ X_{2}& =& \sum _{j=1}^{\ell}b^{j}(0,1,0,\ldots,0) \frac{\partial \ } {\partial x^{j}} + \frac{\partial \ } {\partial x^{\ell+2}} {}\\ & & \qquad \qquad \quad \vdots {}\\ X_{m-\ell}& =& \sum _{j=1}^{\ell}b^{j}(0,0,\ldots,1) \frac{\partial \ } {\partial x^{j}} + \frac{\partial \ } {\partial x^{m}}. {}\\ \end{array}$$

These are C vector fields on U p that span \(\mathcal{H}_{q}\) at each point q ∈ U p . By the subbundle criterion (Theorem  20.4), \(\mathcal{H}\) is a C subbundle of TP. □ 

28.4 Horizontal Lift of a Vector Field to a Principal Bundle

Suppose \(\mathcal{H}\) is a horizontal distribution on a principal bundle π: P → M. Let X be a vector field on M. For every p ∈ P, because the vertical subspace \(\mathcal{V}_{p}\) is kerπ , the differential π : T p P → T π(p) M induces an isomorphism

$$\displaystyle{\frac{T_{p}P} {\ker \pi _{{\ast}}} \mathop{\rightarrow }\limits^{\sim }\mathcal{H}_{p}\mathop{ \rightarrow }\limits^{\sim } T_{\pi (p)}M}$$

of the horizontal subspace \(\mathcal{H}_{p}\) with the tangent space T π(p) M. Consequently, there is a unique horizontal vector \(\tilde{X}_{p} \in \mathcal{H}_{p}\) such that \(\pi _{{\ast}}(\tilde{X}_{p}) = X_{\pi (p)} \in T_{\pi (p)}M\). The vector field \(\tilde{X}\) is called the horizontal lift of X to P.

Proposition 28.6.

If \(\mathcal{H}\) is a C right-invariant horizontal distribution on the total space P of a principal bundle π: P → M, then the horizontal lift \(\tilde{X}\) of a C vector field X on M is a C right-invariant vector field on P.

Proof.

Let x ∈ M and p ∈ π −1(x). By definition, \(\pi _{{\ast}}(\tilde{X}_{p}) = X_{x}\). If q is any other point of π −1(x), then q = pg for some g ∈ G. Since πr g  = π,

$$\displaystyle{\pi _{{\ast}}(r_{g{\ast}}\tilde{X}_{p}) = (\pi \circ r_{g})_{{\ast}}\tilde{X}_{p} =\pi _{{\ast}}\tilde{X}_{p} = X_{p}.}$$

By the uniqueness of the horizontal lift, \(r_{g{\ast}}\tilde{X}_{p} =\tilde{ X}_{pg}\). This proves the right-invariance of \(\tilde{X}\). We prove the smoothness of \(\tilde{X}\) by proving it locally. Let {U} be a trivializing open cover for P with trivializations \(\phi _{U}: \pi ^{-1}(U)\mathop{ \rightarrow }\limits^{\sim } U \times G\). Define

$$\displaystyle{Z_{(x,g)} = (X_{x},0) \in T_{(x,g)}(U \times G).}$$

Let η: U × G → U be the projection to the first factor. Then Z is a C vector field on U × G such that η Z (x, g) = X x , and Y: = (ϕ U)−1 Z is a C vector field on π −1(U) such that π Y p  = X π(p). By Proposition 28.3, hY is a C vector field on π −1(U). Clearly it is horizontal. Because Y p  = v(Y p ) + h(Y p ) and π v(Y p ) = 0, we have π Y p  = π h(Y p ) = X π(p). Thus, hY lifts X over U. By the uniqueness of the horizontal lift, \(hY =\widetilde{ X}\) over U. This proves that \(\tilde{X}\) is a smooth vector field on P. □ 

28.5 Lie Bracket of a Fundamental Vector Field

If a principal bundle P comes with a connection, then it makes sense to speak of horizontal vector fields on P; these are vector fields all of whose vectors are horizontal.

Lemma 28.7.

Suppose P is a principal bundle with a connection. Let A be the fundamental vector field on P associated to \(A \in \mathfrak{g}\).

(i):

If Y is a horizontal vector field on P, then [A ,Y ] is horizontal.

(ii):

If Y is a right-invariant vector field on P, then [A ,Y ] = 0.

Proof.

(i) A local flow for A is \(\phi _{t}(p) = pe^{tA} = r_{e^{tA}}(p)\) (Proposition 27.14). By the identification of the Lie bracket with the Lie derivative of vector fields [21, Th. 20.4, p. 225] and the definition of the Lie derivative,

$$\displaystyle{ [\underline{A},Y ]_{p} = (\mathcal{L}_{\underline{A}}Y )_{p} =\lim _{t\rightarrow 0}\frac{\left (r_{e^{-tA}}\right )_{{\ast}}Y _{pe^{tA}} - Y _{p}} {t}. }$$
(28.5)

Since right translation preserves horizontality (Theorem 28.5), both \(\left (r_{e^{-tA}}\right )_{{\ast}}Y _{pe^{tA}}\) and Y p are horizontal vectors. Denote the difference quotient in (28.5) by c(t). For every t near 0 in \(\mathbb{R}\), c(t) is in the vector space \(\mathcal{H}_{p}\) of horizontal vectors at p. Therefore, \([\underline{A},Y ]_{p} =\lim _{t\rightarrow 0}c(t) \in \mathcal{H}_{p}\).

(ii) If Y is right-invariant, then

$$\displaystyle{(r_{e^{-tA}})_{{\ast}}Y _{pe^{tA}} = Y _{p}.}$$

In that case, it follows from (28.5) that [A, Y ] p  = 0. □ 

Problems

28.1. Maurer–Cartan connection

If θ is the Maurer–Cartan form on a Lie group and π 2: M × G → G is the projection to the second factor, prove that ω: = π 2 θ is a connection on the trivial bundle π 1: M × G → M. It is called the Maurer–Cartan connection.

28.2. Convex linear combinations of connections

Prove that a convex linear combination ω of connections ω 1, , ω n on a principal bundle π: P → M is again a connection on P. (ω = ∑ λ i ω i , ∑ λ i  = 1, λ i  ≥ 0.)

28.3. Pullback of a connection

Let π Q : Q → N and π P : P → M be principal G-bundles, and let \((\bar{f}: Q \rightarrow P,f: N \rightarrow M)\) be a morphism of principal bundles. Prove that if θ is a connection on P, then \(\bar{f}^{{\ast}}\theta\) is a connection on Q.

§29 Horizontal Distributions on a Frame Bundle

In this section we will explain the process by which a connection ∇ on a vector bundle E over a manifold M gives rise to a smooth right-invariant horizontal distribution on the associated frame bundle \(\mathop{\mathrm{Fr}}\nolimits (E)\). This involves a sequence of steps. A connection on the vector bundle E induces a covariant derivative on sections of the vector bundle along a curve. Parallel sections along the curve are those whose derivative vanishes. Just as for tangent vectors in Section 14, starting with a frame e x for the fiber of the vector bundle at the initial point x of a curve, there is a unique way to parallel translate the frame along the curve. In terms of the frame bundle \(\mathop{\mathrm{Fr}}\nolimits (E)\), what this means is that every curve in M has a unique lift to \(\mathop{\mathrm{Fr}}\nolimits (E)\) starting at e x representing parallel frames along the curve. Such a lift is called a horizontal lift. The initial vector at e x of a horizontal lift is a horizontal vector at e x . The horizontal vectors at a point of \(\mathop{\mathrm{Fr}}\nolimits (E)\) form a subspace of the tangent space \(T_{e_{x}}\big(\mathop{\mathrm{Fr}}\nolimits (E)\big)\). In this way we obtain a horizontal distribution on the frame bundle. We show that this horizontal distribution on \(\mathop{\mathrm{Fr}}\nolimits (E)\) arising from a connection on the vector bundle E is smooth and right-invariant. It therefore corresponds to a connection ω on the principal bundle \(\mathop{\mathrm{Fr}}\nolimits (E)\). We then show that ω pulls back under a section e of \(\mathop{\mathrm{Fr}}\nolimits (E)\) to the connection matrix ω e of the connection ∇ relative to the frame e on an open set U.

29.1 Parallel Translation in a Vector Bundle

In Section 14 we defined parallel translation of a tangent vector along a curve in a manifold with an affine connection. In fact, the same development carries over to an arbitrary vector bundle η: E → M with a connection ∇.

Let c: [a, b] → M be a smooth curve in M. Instead of vector fields along the curve c, we consider smooth sections of the pullback bundle c E over [a, b]. These are called smooth sections of the vector bundle E along the curve c. We denote by Γ(c E) the vector space of smooth sections of E along the curve c. If E = TM is the tangent bundle of a manifold M, then an element of Γ(c TM) is simply a vector field along the curve c in M. Just as in Theorem  13.1, there is a unique \(\mathbb{R}\)-linear map

$$\displaystyle{\frac{D\ } {dt}: \varGamma (c^{{\ast}}E) \rightarrow \varGamma (c^{{\ast}}E),}$$

called the covariant derivative corresponding to ∇, such that

(i):

(Leibniz rule) for any C function f on the interval [a, b],

$$\displaystyle{\frac{D(fs)} {dt} = \frac{df} {dt}s + f \frac{Ds} {dt};}$$
(ii):

if s is induced from a global section \(\tilde{s} \in \varGamma (M,E)\) in the sense that \(s(t) =\tilde{ s}\big(c(t)\big)\), then

$$\displaystyle{\frac{Ds} {dt} (t) = \nabla _{c^{\prime}(t)}\tilde{s}.}$$

Definition 29.1.

A section s ∈ Γ(c E) is parallel along a curve c: [a, b] → M if Dsdt ≡ 0 on [a, b].

As in Section 14.5, the equation Dsdt ≡ 0 for a section s to be parallel is equivalent to a system of linear first-order ordinary differential equations. Suppose c: [a, b] → M maps into a framed open set (U, e 1, , e r ) for E. Then s ∈ Γ(c E) can be written as

$$\displaystyle{s(t) =\sum s^{i}(t)e_{ i,c(t)}.}$$

By properties (i) and (ii) of the covariant derivative,

$$\displaystyle\begin{array}{rcl} \frac{Ds} {dt} & =& \sum _{i}\frac{ds^{i}} {dt} e_{i} +\sum _{j}s^{j}\frac{D\ } {dt}e_{j,c(t)} {}\\ & =& \sum _{i}\frac{ds^{i}} {dt} e_{i} +\sum _{j}s^{j}\nabla _{ c^{\prime}(t)}e_{j} {}\\ & =& \sum _{i}\frac{ds^{i}} {dt} e_{i} +\sum _{i,j}s^{j}\omega _{ j}^{i}\big(c^{\prime}(t)\big)e_{ i}. {}\\ \end{array}$$

Hence, Dsdt ≡ 0 if and only if

$$\displaystyle{\frac{ds^{i}} {dt} +\sum _{j}s^{j}\omega _{ j}^{i}\big(c^{\prime}(t)\big) = 0\text{ for all }i.}$$

This is a system of linear first-order differential equations. By the existence and uniqueness theorems of differential equations, it has a solution on a small interval about a give point t 0 and the solution is uniquely determined by its value at t 0. Thus, a parallel section is uniquely determined by its value at a point. If s ∈ Γ(c E) is a parallel section of the pullback bundle c E, we say that s(b) is the parallel transport of s(a) along c: [a, b] → M. The resulting map: E c(a) → E c(b) is called parallel translation from E c(a) to E c(b).

Theorem 29.2.

Let η: E → M be a C vector bundle with a connection ∇ and let c: [a,b] → M be a smooth curve in M. There is a unique parallel translation φ a,b from E c(a) to E c(b) along c. This parallel translation φ a,b : E c(a) → E c(b) is a linear isomorphism.

The proof is similar to that of Theorem 14.14.

A parallel frame along the curve c: [a, b] → M is a collection of parallel sections \(\big(e_{1}(t),\ldots,e_{r}(t)\big)\), t ∈ [a, b], such that for each t, the elements e 1(t), , e r (t) form a basis for the vector space E c(t).

Let \(\pi: \mathop{\mathrm{Fr}}\nolimits (E) \rightarrow M\) be the frame bundle of the vector bundle η: E → M. A curve \(\tilde{c}(t)\) in \(\mathop{\mathrm{Fr}}\nolimits (E)\) is called a lift of the curve c(t) in M if \(c(t) =\pi (\tilde{c}(t))\). It is a horizontal lift if in addition \(\tilde{c}(t)\) is a parallel frame along c.

Restricting the domain of the curve c to the interval [a, t], we obtain from Theorem 29.2 that parallel translation is a linear isomorphism of E c(a) with E c(t). Thus, if a collection of parallel sections \(\big(s_{1}(t),\ldots,s_{r}(t)\big) \in \varGamma (c^{{\ast}}E)\) forms a basis at one time t, then it forms a basis at every time t ∈ [a, b]. By Theorem 29.2, for every smooth curve c: [a, b] → M and ordered basis (s 1, 0, , s r, 0) for E c(a), there is a unique parallel frame along c whose value at a is (s 1, 0, , s r, 0). In terms of the frame bundle \(\mathop{\mathrm{Fr}}\nolimits (E)\), this shows the existence and uniqueness of a horizontal lift with a specified initial point in \(\mathop{\mathrm{Fr}}\nolimits (E)\) of a curve c(t) in M.

29.2 Horizontal Vectors on a Frame Bundle

On a general principal bundle vertical vectors are intrinsically defined, but horizontal vectors are not. We will see shortly that a connection on a vector bundle E over a manifold M determines a well-defined horizontal distribution on the frame bundle \(\mathop{\mathrm{Fr}}\nolimits (E)\). The elements of the horizontal distribution are the horizontal vectors. Thus, the notion of a horizontal vector on the frame bundle \(\mathop{\mathrm{Fr}}\nolimits (E)\) depends on a connection on E.

Definition 29.3.

Let E → M be a vector bundle with a connection ∇, x ∈ M, and \(e_{x} \in \mathop{\mathrm{Fr}}\nolimits (E_{x})\). A tangent vector \(v \in T_{e_{x}}(\mathop{\mathrm{Fr}}\nolimits (E))\) is said to be horizontal if there is a curve c(t) through x in M such that v is the initial vector \(\tilde{c}^{\prime}(0)\) of the unique horizontal lift of \(\tilde{c}(t)\) of c(t) to \(\mathop{\mathrm{Fr}}\nolimits (E)\) starting at e x .

Our goal now is to show that the horizontal vectors at a point e x of the frame bundle form a vector subspace of the tangent space \(T_{e_{x}}\big(\mathop{\mathrm{Fr}}\nolimits (E)\big)\). To this end we will derive an explicit formula for \(\tilde{c}^{\prime}(0)\) in terms of a local frame for E. Suppose c: [0, b] → M is a smooth curve with initial point c(0) = x, and \(\tilde{c}(t)\) is its unique horizontal lift to \(\mathop{\mathrm{Fr}}\nolimits (E)\) with initial point e x  = (e 1, 0, , e r, 0). Let s be a frame for E over a neighborhood U of x with s(x) = e x . Then s(c(t)) is a lift of c(t) to \(\mathop{\mathrm{Fr}}\nolimits (E)\) with initial point e x , but of course it is not necessarily a horizontal lift (see Figure 29.1). For any t ∈ [0, b], we have two ordered bases s(c(t)) and \(\tilde{c}(t)\) for E c(t), so there is a smooth matrix \(a(t) \in \mathop{\mathrm{GL}}\nolimits (r, \mathbb{R})\) such that \(s(c(t)) =\tilde{ c}(t)a(t)\). At t = 0, \(s(c(0)) = e_{x} =\tilde{ c}(0)\), so that a(0) = I, the identity matrix in \(\mathop{\mathrm{GL}}\nolimits (r, \mathbb{R})\).

Fig. 29.1
figure 1

Two liftings of a curve

Lemma 29.4.

In the notation above, let \(s_{{\ast}}: T_{x}(M) \rightarrow T_{e_{x}}(\mathop{\mathrm{Fr}}\nolimits (E))\) be the differential of s and a′(0) the fundamental vector field on \(\mathop{\mathrm{Fr}}\nolimits (E)\) associated to \(a^{\prime}(0) \in \mathfrak{g}\mathfrak{l}(r, \mathbb{R})\). Then

$$\displaystyle{s_{{\ast}}\big(c^{\prime}(0)\big) =\tilde{ c}^{\prime}(0) +\underline{ a^{\prime}(0)}_{e_{x}}.}$$

Proof.

Let \(P =\mathop{ \mathrm{Fr}}\nolimits (E)\) and \(G =\mathop{ \mathrm{GL}}\nolimits (r, \mathbb{R})\), and let μ: P × G → P be the right action of G on P. Then

$$\displaystyle{ s\big(c(t)\big) =\tilde{ c}(t)a(t) =\mu \big (\tilde{c}(t),a(t)\big), }$$
(29.1)

with c(0) = x, \(\tilde{c}(0) = e_{x}\), and a(0) = the identity matrix I. Differentiating (29.1) with respect to t and evaluating at 0 gives

$$\displaystyle{s_{{\ast}}\big(c^{\prime}(0)\big) =\mu _{{\ast},(\tilde{c}(0),a(0))}\big(\tilde{c}^{\prime}(0),a^{\prime}(0)\big).}$$

By the formula for the differential of an action (Problem 27.4),

$$\displaystyle{s_{{\ast}}\big(c^{\prime}(0)\big) = r_{a(0){\ast}}\tilde{c}^{\prime}(0) +\underline{ a^{\prime}(0)}_{\tilde{c}(0)} =\tilde{ c}^{\prime}(0) +\underline{ a^{\prime}(0)}_{e_{x}}.}$$

 □ 

Lemma 29.5.

Let E → M be a vector bundle with a connection ∇. Suppose s = (s 1 ,…,s r ) is a frame for E over an open set U, \(\tilde{c}(t)\) a parallel frame over a curve c(t) in U with \(\tilde{c}(0) = s(c(0))\) , and a(t) the curve in \(\mathop{\mathrm{GL}}\nolimits (r, \mathbb{R})\) such that \(s(c(t)) =\tilde{ c}(t)a(t)\) . If ω s = [ω j i ] is the connection matrix of ∇ with respect to the frame (s 1 ,…,s r ) over U, then a′(0) = ω s (c′(0)).

Proof.

Label c(0) = x and \(\tilde{c}_{i}(0) = s_{i}\big(c(0)\big) = e_{i,x}\). By the definition of the connection matrix,

$$\displaystyle{ \nabla _{c^{\prime}(0)}s_{j} =\sum \omega _{ j}^{i}\big(c^{\prime}(0)\big)s_{ i}\big(c(0)\big) =\sum \omega _{ j}^{i}\big(c^{\prime}(0)\big)e_{ i,x}. }$$
(29.2)

On the other hand, by the defining properties of the covariant derivative (Section 29.1),

$$\displaystyle\begin{array}{rcl} \nabla _{c^{\prime}(t)}s_{j}& =& \frac{D(s_{j} \circ c)} {dt} (t) = \frac{D\ } {dt}\sum \tilde{c}_{i}(t)a_{j}^{i}(t) {}\\ & =& \sum (a_{j}^{i})^{\prime}(t)\tilde{c}_{ i}(t) +\sum a_{j}^{i}(t)\frac{D\tilde{c}_{i}} {dt} (t) {}\\ & =& \sum (a_{j}^{i})^{\prime}(t)\tilde{c}_{ i}(t)\qquad \mbox{$(since D\tilde{c}_{i}/dt \equiv 0)$}. {}\\ \end{array}$$

Setting t = 0 gives

$$\displaystyle{ \nabla _{c^{\prime}(0)}s_{j} =\sum (a_{j}^{i})^{\prime}(0)e_{ i,x}. }$$
(29.3)

Equating (29.2) and (29.3), we obtain (a j i)′(0) = ω j i(c′(0)). □ 

Thus, Lemma 29.4 for the horizontal lift of c′(0) can be rewritten in the form

$$\displaystyle{ \tilde{c}^{\prime}(0) = s_{{\ast}}\big(c^{\prime}(0)\big) -\underline{ a^{\prime}(0)}_{e_{x}} = s_{{\ast}}\big(c^{\prime}(0)\big) -\underline{\omega _{s}\big(c^{\prime}(0)\big)}_{e_{x}}. }$$
(29.4)

Proposition 29.6.

Let π: E → M be a smooth vector bundle with a connection over a manifold M of dimension n. For x ∈ M and e x an ordered basis for the fiber E x , the subset \(\mathcal{H}_{e_{x}}\) of horizontal vectors in the tangent space \(T_{e_{x}}(\mathop{\mathrm{Fr}}\nolimits (E))\) is a vector space of dimension n, and \(\pi _{{\ast}}: \mathcal{H}_{e_{x}} \rightarrow T_{x}M\) is a linear isomorphism.

Proof.

In formula (29.4), ω s (c′(0)) is \(\mathbb{R}\)-linear in its argument c′(0) because ω s is a 1-form at c(0). The operation \(A\mapsto \underline{A}_{e_{x}}\) of associating to a matrix \(A \in \mathfrak{g}\mathfrak{l}(r, \mathbb{R})\) a tangent vector \(\underline{A}_{e_{x}} \in T_{e_{x}}\big(\mathop{\mathrm{Fr}}\nolimits (E)\big)\) is \(\mathbb{R}\)-linear by (27.3). Hence, formula (29.4) shows that the map

$$\displaystyle\begin{array}{rcl} \phi: T_{x}M& \rightarrow & T_{e_{x}}\big(\mathop{\mathrm{Fr}}\nolimits (E)\big), {}\\ c^{\prime}(0)& \mapsto & \tilde{c}^{\prime}(0) {}\\ \end{array}$$

is \(\mathbb{R}\)-linear. As the image of a vector space T x M under a linear map, the set \(\mathcal{H}_{e_{x}}\) of horizontal vectors \(\tilde{c}^{\prime}(0)\) at e x is a vector subspace of \(T_{e_{x}}\big(\mathop{\mathrm{Fr}}\nolimits (E)\big)\).

Since \(\pi \big(\tilde{c}(t)\big) = c(t)\), taking the derivative at t = 0 gives \(\pi _{{\ast}}\big(\tilde{c}^{\prime}(0)\big) = c^{\prime}(0)\), so π is a left inverse to the map ϕ. This proves that \(\phi: T_{x}M \rightarrow T_{e_{x}}\big(\mathop{\mathrm{Fr}}\nolimits (E)\big)\) is injective. Its image is by definition \(\mathcal{H}_{e_{x}}\). It follows that \(\phi: T_{x}M \rightarrow \mathcal{H}_{e_{x}}\) is an isomorphism with inverse \(\pi _{{\ast}}: \mathcal{H}_{e_{x}} \rightarrow T_{e_{x}}M\). □ 

29.3 Horizontal Lift of a Vector Field to a Frame Bundle

We have learned so far that a connection on a vector bundle E → M defines a horizontal subspace \(\mathcal{H}_{p}\) of the tangent space T p P at each point p of the total space of the frame bundle \(\pi: P =\mathop{ \mathrm{Fr}}\nolimits (E) \rightarrow M\). The horizontal subspace \(\mathcal{H}_{p}\) has the same dimension as M. The vertical subspace \(\mathcal{V}_{p}\) of T p P is the kernel of the surjection π : T p P → T π(p) M; as such, \(\dim \mathcal{V}_{p} =\dim T_{p}P -\dim M\). Hence, \(\mathcal{V}_{p}\) and \(\mathcal{H}_{p}\) have complementary dimensions in T p P. Since \(\pi _{{\ast}}(\mathcal{V}_{p}) = 0\) and \(\pi _{{\ast}}: \mathcal{H}_{p} \rightarrow T_{\pi (p)}M\) is an isomorphism, \(\mathcal{V}_{p} \cap \mathcal{H}_{p} = 0\). It follows that there is a direct sum decomposition

$$\displaystyle{ T_{p}(\mathop{\mathrm{Fr}}\nolimits (E)) = \mathcal{V}_{p} \oplus \mathcal{H}_{p}. }$$
(29.5)

Our goal now is to show that as p varies in P, the subset \(\mathcal{H}:=\bigcup _{p\in P}\mathcal{H}_{p}\) of the tangent bundle TP defines a C horizontal distribution on P in the sense of Section 27.6.

Since \(\pi _{{\ast},p}: \mathcal{H}_{p} \rightarrow T_{\pi (p)}M\) is an isomorphism for each p ∈ P, if X is a vector field on M, then there is a unique vector field \(\tilde{X}\) on P such that \(\tilde{X}_{p} \in \mathcal{H}_{p}\) and \(\pi _{{\ast},p}(\tilde{X}_{p}) = X_{\pi (p)}\). The vector field \(\tilde{X}\) is called the horizontal lift of X to the frame bundle P.

Since every tangent vector X x  ∈ T x M is the initial vector c′(0) of a curve c, formula (29.4) for the horizontal lift of a tangent vector can be rewritten in the following form.

Lemma 29.7 (Horizontal lift formula).

Suppose ∇ is a connection on a vector bundle E → M and ω s is its connection matrix on a framed open set (U,s). For x ∈ U, \(p = s(x) \in \mathop{\mathrm{Fr}}\nolimits (E)\) , and X x ∈ T x M, let \(\tilde{X}_{p}\) be the horizontal lift of X x to p in \(\mathop{\mathrm{Fr}}\nolimits (E)\) . Then

$$\displaystyle{\tilde{X}_{p} = s_{{\ast},x}(X_{x}) -\underline{\omega _{s}(X_{x})}_{p}.}$$

Proposition 29.8.

Let E → M be a C rank r vector bundle with a connection and \(\pi: \mathop{\mathrm{Fr}}\nolimits (E) \rightarrow M\) its frame bundle. If X is a C vector field on M, then its horizontal lift \(\tilde{X}\) to \(\mathop{\mathrm{Fr}}\nolimits (E)\) is a C vector field.

Proof.

Let \(P =\mathop{ \mathrm{Fr}}\nolimits (E)\) and \(G =\mathop{ \mathrm{GL}}\nolimits (r, \mathbb{R})\). Since the question is local, we may assume that the bundle P is trivial, say P = M × G. By the right invariance of the horizontal distribution,

$$\displaystyle{ \tilde{X}_{(x,a)} = r_{a{\ast}}\tilde{X}_{(x,1)}. }$$
(29.6)

Let s: M → P = M × G be the section s(x) = (x, 1). By the horizontal lift formula (Lemma 29.7),

$$\displaystyle{ \tilde{X}_{(x,1)} = s_{{\ast},x}(X_{x}) -\underline{\omega _{s}(X_{x})}_{(x,1)}. }$$
(29.7)

Let p = (x, a) ∈ P and let f be a C function on P. We will prove that \(\tilde{X}_{p}f\) is C as a function of p. By (29.6) and (29.7),

$$\displaystyle{ \tilde{X}_{p}f = r_{a{\ast}}s_{{\ast},x}(X_{x})f - r_{a{\ast}}\underline{\omega _{s}(X_{x})}_{(x,1)}f, }$$
(29.8)

so it suffices to prove separately that \(\big(r_{a{\ast}}(s_{{\ast},x}X_{x})\big)f\) and \(\big(r_{a{\ast}}\underline{\omega _{s}(X_{x})}_{(x,1)}\big)f\) are C functions on P.

The first term is

$$\displaystyle\begin{array}{rcl} \big(r_{a{\ast}}s_{{\ast},x}(X_{x})\big)f& =& X_{x}(f \circ r_{a} \circ s) \\ & =& X(f \circ r_{a} \circ s)\big(\pi (p)\big) \\ & =& X\big(f\big(s(\pi (p))a\big)\big) = X\big(f\big(\mu (s(\pi (p)),a)\big)\big) \\ & =& X\big(f\big(\mu (s(\pi (p)),\pi _{2}(p))\big)\big), {}\end{array}$$
(29.9)

where μ: P × G → P is the action of G on P and π 2: P = M × G → G is the projection π 2(p) = π 2(x, a) = a. The formula (29.9) expresses \(\big(r_{a{\ast}}s_{{\ast},x}(X_{x})\big)f\) as a C function on P.

By the right equivariance of the connection form ω s , in (29.8) the second term can be rewritten as

$$\displaystyle\begin{array}{rcl} r_{a{\ast}}\underline{\omega _{s}(X_{x})}_{(x,1)}f& =& \underline{(\mathop{\mathrm{Ad}}\nolimits a^{-1})\omega _{ s}(X_{x})}_{(x,a)}f {}\\ & =& \underline{\big(\mathop{\mathrm{Ad}}\nolimits \pi _{2}(p)^{-1}\big)\omega _{ s}\big(X_{\pi (p)}\big)}_{p}f, {}\\ \end{array}$$

where \(\big(\mathop{\mathrm{Ad}}\nolimits \pi _{2}(p)^{-1}\big)\omega _{s}(X_{\pi (p)})\) is a C function: \(P \rightarrow \mathfrak{g}\mathfrak{l}(r, \mathbb{R})\) that we will denote by A(p). The problem now is to show that pA(p) p f is a C function of p.

Let μ: P × G → P be the right action of \(G =\mathop{ \mathrm{GL}}\nolimits (r, \mathbb{R})\) on \(P =\mathop{ \mathrm{Fr}}\nolimits (E)\). Then

$$\displaystyle{\underline{A(p)}_{p}f = \left. \frac{d} {dt}\right \vert _{t=0}f(p \cdot e^{tA(p)}) = \left. \frac{d} {dt}\right \vert _{t=0}f(\mu (p,e^{tA(p)})).}$$

Since f, μ, A, and the exponential map are all C functions, A(p) p f is a C function of p. Thus, \(\tilde{X}_{p}f\) in (29.8) is a C function of p. This proves that \(\tilde{X}\) is a C vector field on P. □ 

Theorem 29.9.

A connection ∇ on a smooth vector bundle E → M defines a C distribution \(\mathcal{H}\) on the frame bundle \(\pi: P =\mathop{ \mathrm{Fr}}\nolimits (E) \rightarrow M\) such that at any p ∈ P,

(i):

\(T_{p}P = \mathcal{V}_{p} \oplus \mathcal{H}_{p}\);

(ii):

\(r_{g{\ast}}(\mathcal{H}_{p}) = \mathcal{H}_{pg}\) for any \(g \in G =\mathop{ \mathrm{GL}}\nolimits (r, \mathbb{R})\),

where r g : P → P is the right action of G on P.

Proof.

To prove that \(\mathcal{H}\) is a C subbundle of TP, let U be a coordinate open set in M and s 1, , s n a C frame on U. By Proposition 29.8 the horizontal lifts \(\widetilde{s_{1}},\ldots,\widetilde{s_{n}}\) are C vector fields on \(\tilde{U}:=\pi ^{-1}(U)\). Moreover, for each \(p \in \tilde{ U}\), since \(\pi _{{\ast},p}: \mathcal{H}_{p} \rightarrow T_{\pi (p)}M\) is an isomorphism, \((\widetilde{s_{1}})_{p},\ldots,(\widetilde{s_{n}})_{p}\) form a basis for \(\mathcal{H}_{p}\). Thus, over \(\tilde{U}\) the C sections \(\widetilde{s_{1}},\ldots,\widetilde{s_{n}}\) of TP span \(\mathcal{H}\). By Theorem  20.4, this proves that \(\mathcal{H}\) is a C subbundle of TP.

Equation (29.5) establishes (i).

As for (ii), let \(\tilde{c}^{\prime}(0) \in \mathcal{H}_{p}\), where c(t) is a curve in M and \(\tilde{c}(t) = [v_{1}(t)\ \cdots \ v_{r}(t)]\) is its horizontal lift to P with initial point p. Here we are writing a frame as a row vector so that the group action is simply matrix multiplication on the right. For any \(g = [g_{j}^{i}] \in \mathop{\mathrm{GL}}\nolimits (r, \mathbb{R})\),

$$\displaystyle{\tilde{c}(t)g = \left [\sum g_{1}^{i}v_{ i}(t)\ \cdots \ \sum g_{r}^{i}v_{ i}(t)\right ].}$$

Since Dv i dt ≡ 0 by the horizontality of v i and g j i are constants, D(∑ g j i v i )∕dt ≡ 0. Thus, \(\tilde{c}(t)g\) is the horizontal lift of c(t) with initial point \(\tilde{c}(0)g\). It has initial tangent vector

$$\displaystyle{\left. \frac{d} {dt}\right \vert _{t=0}\tilde{c}(t)g = r_{g{\ast}}\tilde{c}^{\prime}(0) \in \mathcal{H}_{pg}.}$$

This proves that \(r_{g{\ast}}\mathcal{H}_{p} \subset \mathcal{H}_{pg}\). Because \(r_{g{\ast}}: \mathcal{H}_{p} \rightarrow \mathcal{H}_{pg}\) has a two-sided inverse r g −1 , it is bijective. In particular, \(r_{g{\ast}}\mathcal{H}_{p} = \mathcal{H}_{pg}\). □ 

29.4 Pullback of a Connection on a Frame Bundle Under a Section

Recall that a connection ∇ on a vector bundle E can be represented on a framed open set (U, e 1, , e r ) for E by a connection matrix ω e depending on the frame. Such a frame e = (e 1, , e r ) is in fact a section \(e: U \rightarrow \mathop{\mathrm{Fr}}\nolimits (E)\) of the frame bundle. We now use the horizontal lift formula (Lemma 29.7) to prove that the Ehresmann connection ω on the frame bundle \(\mathop{\mathrm{Fr}}\nolimits (E)\) determined by ∇ pulls back under the section e to the connection matrix ω e .

Theorem 29.10.

Let ∇ be a connection on a vector bundle E → M and let ω be the Ehresmann connection on the frame bundle \(\mathop{\mathrm{Fr}}\nolimits (E)\) determined by ∇. If e = (e 1 ,…,e r ) is a frame for E over an open set U, viewed as a section \(e: U \rightarrow \mathop{\mathrm{Fr}}\nolimits (E)\vert _{U}\) , and ω e is the connection matrix of ∇ relative to the frame e, then ω e = e ω.

Proof.

Let x ∈ U and \(p = e(x) \in \mathop{\mathrm{Fr}}\nolimits (E)\). Suppose X x is a tangent vector to M at x. If we write ω e, x for the value of the connection matrix ω e at the point x ∈ U, then ω e, x is an r × r matrix of 1-forms at x and ω e, x (X x ) is an r × r matrix of real numbers, i.e., an element of the Lie algebra \(\mathfrak{g}\mathfrak{l}(r, \mathbb{R})\). The corresponding fundamental vector field on \(\mathop{\mathrm{Fr}}\nolimits (E)\) is ω e, x (X x ). By Lemma 29.7, the horizontal lift of X x to \(p \in \mathop{\mathrm{Fr}}\nolimits (E)\) is

$$\displaystyle{\tilde{X}_{p} = e_{{\ast}}X_{x} -\underline{\omega _{e,x}(X_{x})}_{p}.}$$

Applying the Ehresmann connection ω p to both sides of this equation, we get

$$\displaystyle\begin{array}{rcl} 0& =& \omega _{p}(\tilde{X}_{p}) =\omega _{p}(e_{{\ast}}X_{x}) -\omega _{p}\left (\underline{\omega _{e,x}(X_{x})}_{p}\right ) {}\\ & =& (e^{{\ast}}\omega _{ p})(X_{x}) -\omega _{e,x}(X_{x})\quad \mathrm{(by Theorem\ 28.1(i))}. {}\\ \end{array}$$

Since this is true for all X x  ∈ T x M,

$$\displaystyle{e^{{\ast}}\omega _{ p} = (e^{{\ast}}\omega )_{ x} =\omega _{e,x}.}$$

 □ 

§30 Curvature on a Principal Bundle

Let G be a Lie group with Lie algebra \(\mathfrak{g}\). Associated to a connection ω on a principal G-bundle is a \(\mathfrak{g}\)-valued 2-form Ω called its curvature. The definition of the curvature is suggested by the second structural equation for a connection ∇ on a vector bundle E. Just as the connection form ω on the frame bundle \(\mathop{\mathrm{Fr}}\nolimits (E)\) pulls back by a section e of \(\mathop{\mathrm{Fr}}\nolimits (E)\) to the connection matrix ω e of ∇ with respect to the frame e, so the curvature form Ω on the frame bundle \(\mathop{\mathrm{Fr}}\nolimits (E)\) pulls back by e to the curvature matrix Ω e of ∇ with respect to e. Thus, the curvature form Ω on the frame bundle is an intrinsic object of which the curvature matrices Ω e are but local manifestations.

30.1 Curvature Form on a Principal Bundle

By Theorem  11.1 if ∇ is a connection on a vector bundle E → M, then its connection and curvature matrices ω e and Ω e on a framed open set (U, e) = (U, e 1, , e r ) are related by the second structural equation (Theorem  11.1)

$$\displaystyle{\varOmega _{e} = d\omega _{e} +\omega _{e} \wedge \omega _{e}.}$$

In terms of the Lie bracket of matrix-valued forms (see (21.12)), this can be rewritten as

$$\displaystyle{\varOmega _{e} = d\omega _{e} + \frac{1} {2}[\omega _{e},\omega _{e}].}$$

An Ehresmann connection on a principal bundle is Lie algebra-valued. In a general Lie algebra, the wedge product is not defined, but the Lie bracket is always defined. This strongly suggests the following definition for the curvature of an Ehresmann connection on a principal bundle.

Definition 30.1.

Let G be a Lie group with Lie algebra \(\mathfrak{g}\). Suppose ω is an Ehresmann connection on a principal G-bundle π: P → M. Then the curvature of the connection ω is the \(\mathfrak{g}\)-valued 2-form

$$\displaystyle{\varOmega = d\omega + \frac{1} {2}[\omega,\omega ].}$$

Recall that frames for a vector bundle E over an open set U are sections of the frame bundle \(\mathop{\mathrm{Fr}}\nolimits (E)\). Let ω be the connection form on the frame bundle \(\mathop{\mathrm{Fr}}\nolimits (E)\) determined by a connection ∇ on E. In the same way that ω pulls back by sections of \(\mathop{\mathrm{Fr}}\nolimits (E)\) to connection matrices, the curvature form Ω of the connection ω on \(\mathop{\mathrm{Fr}}\nolimits (E)\) pulls back by sections to curvature matrices.

Proposition 30.2.

If ∇ is a connection on a vector bundle E → M and ω is the associated Ehresmann connection on the frame bundle \(\mathop{\mathrm{Fr}}\nolimits (E)\) , then the curvature matrix Ω e relative to a frame e = (e 1 ,…,e r ) for E over an open set U is the pullback e Ω of the curvature Ω on the frame bundle \(\mathop{\mathrm{Fr}}\nolimits (E)\).

Proof.

$$\displaystyle\begin{array}{rcl} e^{{\ast}}\varOmega & =& e^{{\ast}}d\omega + \frac{1} {2}e^{{\ast}}[\omega,\omega ] {}\\ & =& de^{{\ast}}\omega + \frac{1} {2}[e^{{\ast}}\omega,e^{{\ast}}\omega ]\quad \mbox{ ($e^{{\ast}}$ commutes with $d$ and $[,]$ by Proposition\ 21.8)} {}\\ & =& d\omega _{e} + \frac{1} {2}[\omega _{e},\omega _{e}]\qquad\ \ \mathrm{(by Theorem\ 29.10)} {}\\ & =& \varOmega _{e}.\qquad\qquad\qquad\qquad \text{(by the second structural equation)} {}\\ \end{array}$$

 □ 

30.2 Properties of the Curvature Form

Now that we have defined the curvature of a connection on a principal G-bundle π: P → M, it is natural to study some of its properties. Like a connection form, the curvature form Ω is equivariant with respect to right translation on P and the adjoint representation on \(\mathfrak{g}\). However, unlike a connection form, a curvature form is horizontal in the sense that it vanishes as long as one argument is vertical. In this respect it acts almost like the opposite of a connection form, which vanishes on horizontal vectors.

Lemma 30.3.

Let G be a Lie group with Lie algebra \(\mathfrak{g}\) and π: P → M a principal G-bundle with a connection ω. Fix a point p ∈ P.

(i):

Every vertical vector X p ∈ T p P can be extended to a fundamental vector field A on P for some \(A \in \mathfrak{g}\).

(ii):

Every horizontal vector Y p ∈ T p P can be extended to the horizontal lift \(\tilde{B}\) of a C vector field B on M.

Proof.

(i) By the surjectivity of \(j_{p{\ast}}: \mathfrak{g} \rightarrow \mathcal{V}_{p}\) (Proposition 27.18) and Equation (27.3),

$$\displaystyle{X_{p} = j_{p{\ast}}(A) =\underline{ A}_{p}}$$

for some \(A \in \mathfrak{g}\). Then the fundamental vector field A on P extends X p .

(ii) Let x = π(p) in M and let B x be the projection π (Y p ) ∈ T x M of the vector Y p . We can extend B x to a smooth vector field B on M. The horizontal lift \(\tilde{B}\) of B extends Y p on P. □ 

By Proposition 28.6, such a horizontal lift \(\tilde{B}\) is necessarily right-invariant.

Theorem 30.4.

Let G be a Lie group with Lie algebra \(\mathfrak{g}\) . Suppose π: P → M is a principal G-bundle, ω a connection on P, and Ω the curvature form of ω.

(i):

(Horizontality) For p ∈ P and X p ,Y p ∈ T p P,

$$\displaystyle{ \varOmega _{p}(X_{p},Y _{p}) = (d\omega )_{p}(hX_{p},hY _{p}). }$$
(30.1)
(ii):

(G-equivariance) For g ∈ G, we have \(r_{g}^{{\ast}}\varOmega = (\mathop{\mathrm{Ad}}\nolimits g^{-1})\varOmega\).

(iii):

(Second Bianchi identity) dΩ = [Ω,ω].

Proof.

(i) Since both sides of (30.1) are linear in X p and in Y p , we may decompose X p and Y p into vertical and horizontal components, and so it suffices to check the equation for vertical and horizontal vectors only. There are three cases.

Case 1. Both X p and Y p are horizontal. Then

$$\displaystyle\begin{array}{rcl} \varOmega _{p}(X_{p},Y _{p})& =& (d\omega )_{p}(X_{p},Y _{p}) + \frac{1} {2}[\omega _{p},\omega _{p}](X_{p},Y _{p})\qquad \qquad \quad \mbox{(definition of $\varOmega$ )} {}\\ & =& (d\omega )_{p}(X_{p},Y _{p}) {}\\ & & \ \ +\frac{1} {2}\Big([\omega _{p}(X_{p}),\omega _{p}(Y _{p})] - [\omega _{p}(Y _{p}),\omega _{p}(X_{p})]\Big) {}\\ & =& (d\omega )_{p}(X_{p},Y _{p})\qquad \qquad \qquad \qquad \qquad \qquad \quad \ \mbox{ ($\omega _{p}(X_{p}) = 0$)} {}\\ & =& (d\omega )_{p}(hX_{p},hY _{p}).\qquad \qquad \qquad \qquad \qquad \qquad \mbox{ ($X_{p}$, $Y _{p}$ horizontal)} {}\\ \end{array}$$

Case 2. One of X p and Y p is horizontal; the other is vertical. Without loss of generality, we may assume X p vertical and Y p horizontal. Then [ω p , ω p ](X p , Y p ) = 0 as in Case 1.

By Lemma 30.3 the vertical vector X p extends to a fundamental vector field A on P and the horizontal vector Y p extends to a right-invariant horizontal vector field \(\tilde{B}\) on P. By the global formula for the exterior derivative (Problem 21.105)

$$\displaystyle{d\omega (\underline{A},\tilde{B}) =\underline{ A}(\omega (\tilde{B})) -\tilde{ B}(\omega (\underline{A})) -\omega ([\underline{A},\tilde{B}]).}$$

On the right-hand side, \(\omega (\tilde{B}) = 0\) because \(\tilde{B}\) is horizontal, and \(\tilde{B}\omega (\underline{A}) =\tilde{ B}A = 0\) because A is a constant function on P. Being the bracket of a fundamental and a horizontal vector field, \([\underline{A},\tilde{B}]\) is horizontal by Lemma 28.7, and therefore \(\omega([\underline{A}, \tilde{B}])=0\), the left-hand side of (30.1) becomes

$$\displaystyle{\varOmega _{p}(X_{p},Y _{p}) = (d\omega )_{p}(\underline{A}_{p},\tilde{B}_{p}) = 0.}$$

The right-hand side of (30.1) is also zero because hX p  = 0.

Case 3. Both X p and Y p are vertical. As in Case 2, we can write X p  = A p and Y p  = B p for some \(A,B \in \mathfrak{g}\). We have thus extended the vertical vectors X p and Y p to fundamental vector fields X = A and Y = B on P. By the definition of curvature,

$$\displaystyle\begin{array}{rcl} \varOmega (X,Y )& =& \varOmega (\underline{A},\underline{B}) \\ & =& d\omega (\underline{A},\underline{B}) + \frac{1} {2}\big([\omega (\underline{A}),\omega (\underline{B})] - [\omega (\underline{B}),\omega (\underline{A})]\big) \\ & =& d\omega (\underline{A},\underline{B}) + [A,B]. {}\end{array}$$
(30.2)

In this sum the first term is

$$\displaystyle\begin{array}{rcl} d\omega (\underline{A},\underline{B})& =& \underline{A}\big(\omega (\underline{B})\big) -\underline{ B}\big(\omega (\underline{A})\big) -\omega \big ([\underline{A},\underline{B}]\big) {}\\ & =& \underline{A}(B) -\underline{ B}(A) -\omega \big (\underline{[A,B]}\big)\qquad \qquad \mathrm{(Problem\ 27.1)} {}\\ & =& 0 - 0 - [A,B]. {}\\ \end{array}$$

Hence, (30.2) becomes

$$\displaystyle{\varOmega (X,Y ) = -[A,B] + [A,B] = 0.}$$

On the other hand,

$$\displaystyle{(d\omega )_{p}(hX_{p},hY _{p}) = (d\omega )_{p}(0,0) = 0.}$$

(ii) Since the connection form ω is right-equivariant with respect to \(\mathop{\mathrm{Ad}}\nolimits\),

$$\displaystyle\begin{array}{rcl} r_{g}^{{\ast}}\varOmega & =& r_{ g}^{{\ast}}\left (d\omega + \frac{1} {2}[\omega,\omega ]\right )\qquad \qquad \qquad \qquad \qquad \text{(definition of curvature)} {}\\ & =& dr_{g}^{{\ast}}\omega + \frac{1} {2}[r_{g}^{{\ast}}\omega,r_{ g}^{{\ast}}\omega ]\qquad \qquad \qquad \qquad \qquad \mathrm{(Proposition\ 21.8)} {}\\ & =& d(\mathop{\mathrm{Ad}}\nolimits g^{-1})\omega + \frac{1} {2}[(\mathop{\mathrm{Ad}}\nolimits g^{-1})\omega,(\mathop{\mathrm{Ad}}\nolimits g^{-1})\omega ] {}\\ & =& (\mathop{\mathrm{Ad}}\nolimits g^{-1})\left (d\omega + \frac{1} {2}[\omega,\omega ]\right ) {}\\ & =& (\mathop{\mathrm{Ad}}\nolimits g^{-1})\varOmega. {}\\ \end{array}$$

In this computation we used the fact that because \(\mathop{\mathrm{Ad}}\nolimits g^{-1} = (c_{g^{-1}})_{{\ast}}\) is the differential of a Lie group homomorphism, it is a Lie algebra homomorphism.

(iii) Taking the exterior derivative of the definition of the curvature form, we get

$$\displaystyle\begin{array}{rcl} d\varOmega & =& \frac{1} {2}d[\omega,\omega ] {}\\ & =& \frac{1} {2}([d\omega,\omega ] - [\omega,d\omega ])\qquad \qquad \qquad \mathrm{(Proposition\ 21.6)} {}\\ & =& [d\omega,\omega ]\quad \quad \qquad \qquad \qquad \qquad \ \quad \mathrm{(Proposition\ 21.5)} {}\\ & =& {\Bigl [\varOmega -\frac{1} {2}[\omega,\omega ],\omega \Bigr ]}\qquad \qquad \qquad \quad \mbox{(definition of $\varOmega$ )} {}\\ & =& [\varOmega,\omega ] -\frac{1} {2}[[\omega,\omega ],\omega ] {}\\ & =& [\varOmega,\omega ].\ \quad \qquad \qquad \qquad \qquad \qquad \mathrm{(Problem\ 21.102)} {}\\ \end{array}$$

 □ 

In case P is the frame bundle \(\mathop{\mathrm{Fr}}\nolimits (E)\) of a rank r vector bundle E, with structure group \(\mathop{\mathrm{GL}}\nolimits (r, \mathbb{R})\), the second Bianchi identity becomes by Proposition  21.7

$$\displaystyle{ d\varOmega = [\varOmega,\omega ] =\varOmega \wedge \omega -\omega \wedge \varOmega, }$$
(30.3)

where the connection and curvature forms ω and Ω are \(\mathfrak{g}\mathfrak{l}(r, \mathbb{R})\)-valued forms on \(\mathop{\mathrm{Fr}}\nolimits (E)\). It should not be so surprising that it has the same form as the second Bianchi identity for the connection and curvature matrices relative to a frame e for E (Proposition  22.3). Indeed, by pulling back (30.3) by a frame \(e: U \rightarrow \mathop{\mathrm{Fr}}\nolimits (E)\), we get

$$\displaystyle\begin{array}{rcl} e^{{\ast}}d\varOmega & =& e^{{\ast}}(\varOmega \wedge \omega ) - e^{{\ast}}(\omega \wedge \varOmega ), {}\\ de^{{\ast}}\varOmega & =& (e^{{\ast}}\varOmega ) \wedge e^{{\ast}}\omega - (e^{{\ast}}\omega ) \wedge e^{{\ast}}\varOmega, {}\\ d\varOmega _{e}& =& \varOmega _{e} \wedge \omega _{e} -\omega _{e} \wedge \varOmega _{e}, {}\\ \end{array}$$

which is precisely Proposition  22.3.

Problems

30.1. Curvature of the Maurer–Cartan connection

Let G be a Lie group with Lie algebra \(\mathfrak{g}\), and M a manifold. Compute the curvature of the Maurer–Cartan connection ω on the trivial bundle π: M × G → M.

30.2. Generalized second Bianchi identity on a frame bundle

Suppose \(\mathop{\mathrm{Fr}}\nolimits (E)\) is the frame bundle of a rank r vector bundle E over M. Let ω be an Ehresmann connection and Ω its curvature form on \(\mathop{\mathrm{Fr}}\nolimits (E)\). These are differential forms on \(\mathop{\mathrm{Fr}}\nolimits (E)\) with values in the Lie algebra \(\mathfrak{g}\mathfrak{l}(r, \mathbb{R})\). Matrix multiplication and the Lie bracket on \(\mathfrak{g}\mathfrak{l}(r, \mathbb{R})\) lead to two ways to multiply \(\mathfrak{g}\mathfrak{l}(r, \mathbb{R})\)-valued forms (see Section 21.5). We write Ω k to denote the wedge product of Ω with itself k times. Prove that d(Ω k) = [Ω k, ω].

30.3. Lie bracket of horizontal vector fields

Let P → M be a principal bundle with a connection, and X, Y horizontal vector fields on P.

(a):

Prove that Ω(X, Y ) = −ω([X, Y ]).

(b):

Show that [X, Y ] is horizontal if and only if the curvature Ω(X, Y ) equals zero.

§31 Covariant Derivative on a Principal Bundle

Throughout this chapter, G will be a Lie group with Lie algebra \(\mathfrak{g}\) and V will be a finite-dimensional vector space. To a principal G-bundle π: P → M and a representation \(\rho: G \rightarrow \mathop{\mathrm{GL}}\nolimits (V )\), one can associate a vector bundle P × ρ V → M with fiber V. When ρ is the adjoint representation \(\mathop{\mathrm{Ad}}\nolimits\) of G on its Lie algebra \(\mathfrak{g}\), the associated bundle \(P \times _{\mathop{\mathrm{Ad}}\nolimits }\mathfrak{g}\) is called the adjoint bundle, denoted by \(\mathop{\mathrm{Ad}}\nolimits P\).

Differential forms on M with values in the associated bundle P × ρ V turn out to correspond in a one-to-one manner to certain V -valued forms on P called tensorial forms of type ρ. The curvature Ω of a connection ω on the principal bundle P is a \(\mathfrak{g}\)-valued tensorial 2-form of type \(\mathop{\mathrm{Ad}}\nolimits\) on P. Under this correspondence it may be viewed as a 2-form on M with values in the adjoint bundle \(\mathop{\mathrm{Ad}}\nolimits P\).

Using a connection ω, one can define a covariant derivative D of vector-valued forms on a principal bundle P. This covariant derivative maps tensorial forms to tensorial forms, and therefore induces a covariant derivative on forms on M with values in an associated bundle. In terms of the covariant derivative D, the curvature form is Ω = D ω, and Bianchi’s second identity becomes D Ω = 0.

31.1 The Associated Bundle

Let π: P → M be a principal G-bundle and \(\rho: G \rightarrow \mathop{\mathrm{GL}}\nolimits (V )\) a representation of G on a finite-dimensional vector space V. We write ρ(g)v as g ⋅ v or even gv. The associated bundle E: = P × ρ V is the quotient of P × V by the equivalence relation

$$\displaystyle{ (p,v) \sim (pg,g^{-1} \cdot v)\qquad \mbox{ for $g \in G$ and $(p,v) \in P \times V $}. }$$
(31.1)

We denote the equivalence class of (p, v) by [p, v]. The associated bundle comes with a natural projection β: P × ρ V → M, β([p, v]) = π(p). Because

$$\displaystyle{\beta ([pg,g^{-1} \cdot v]) =\pi (pg) =\pi (p) =\beta ([p,v]),}$$

the projection β is well defined.

As a first example, the proposition below shows that an associated bundle of a trivial principal G-bundle is a trivial vector bundle.

Proposition 31.1.

If \(\rho: G \rightarrow \mathop{\mathrm{GL}}\nolimits (V )\) is a finite-dimensional representation of a Lie group G, and U is any manifold, then there is a fiber-preserving diffeomorphism

$$\displaystyle\begin{array}{rcl} & & \phi: (U \times G) \times _{\rho }V \mathop{ \rightarrow }\limits^{\sim } U \times V, {}\\ & & \qquad \quad \ [(x,g),v]\mapsto (x,g \cdot v). {}\\ \end{array}$$

Proof.

The map ϕ is well defined because if h is any element of G, then

$$\displaystyle{\phi \big([(x,g)h,h^{-1} \cdot v]\big) =\big (x,(gh) \cdot h^{-1} \cdot v\big) = (x,g \cdot v) =\phi \big ([(x,g),v]\big).}$$

Define ψ: U × V → (U × G) × ρ V by

$$\displaystyle{\psi (x,v) = [(x,1),v].}$$

It is easy to check that ϕ and ψ are inverse to each other, are C , and commute with the projections. □ 

Since a principal bundle P → M is locally U × G, Proposition 31.1 shows that the associated bundle P × ρ V → M is locally trivial with fiber V. The vector space structure on V then makes P × ρ V into a vector bundle over M:

$$\displaystyle{ \begin{array}{rcl} [p,v_{1}] + [p,v_{2}]& =&[p,v_{1} + v_{2}], \\ \lambda [p,v]& =&[p,\lambda v],\quad \lambda \in \mathbb{R}.\end{array} }$$
(31.2)

It is easy to show that these are well-defined operations not depending on the choice of p ∈ E x and that this makes the associated bundle β: E → M into a vector bundle (Problem 31.2).

Example 31.2.

Let \(\mathop{\mathrm{Ad}}\nolimits: G \rightarrow \mathop{\mathrm{GL}}\nolimits (\mathfrak{g})\) be the adjoint representation of a Lie group G on its Lie algebra \(\mathfrak{g}\). For a principal G-bundle π: P → M, the associated vector bundle \(\mathop{\mathrm{Ad}}\nolimits P:= P \times _{\mathop{\mathrm{Ad}}\nolimits }\mathfrak{g}\) is called the adjoint bundle of P.

31.2 The Fiber of the Associated Bundle

If π: P → M is a principal G-bundle, \(\rho: G \rightarrow \mathop{\mathrm{GL}}\nolimits (V )\) is a representation, and E: = P × ρ V → M is the associated bundle, we denote by P x the fiber of P above x ∈ M, and by E x the fiber of E above x ∈ M. For each p ∈ P x , there is a canonical way of identifying the fiber E x with the vector space V:

$$\displaystyle\begin{array}{rcl} f_{p}: V & \rightarrow & E_{x}, {}\\ v& \mapsto & [p,v]. {}\\ \end{array}$$

Lemma 31.3.

Let π: P → M be a principal G-bundle, \(\rho: G \rightarrow \mathop{\mathrm{GL}}\nolimits (V )\) a finite-dimensional representation, and E = P × ρ V the associated vector bundle. For each point p in the fiber P x , the map f p : V → E x is a linear isomorphism.

Proof.

Suppose [p, v] = [p, w]. Then (p, w) = (pg, g −1 v) for some g ∈ G. Since G acts freely on P, the equality p = pg implies that g = 1. Hence, w = g −1 v = v. This proves that f p is injective.

If [q, w] is any point in E x , then q ∈ P x , so q = pg for some g ∈ G. It follows that

$$\displaystyle{[q,w] = [pg,w] = [p,gw] = f_{p}(gw).}$$

This proves that f p is surjective. □ 

The upshot is that every point p of the total space P of a principal bundle gives a linear isomorphism f p : V → E π(p) from V to the fiber of the associated bundle E above π(p).

Lemma 31.4.

Let E = P × ρ V be the vector bundle associated to the principal G-bundle P → M via the representation \(\rho: G \rightarrow \mathop{\mathrm{GL}}\nolimits (V )\) , and f p : V → E x the linear isomorphism v ↦ [p,v]. If g ∈ G, then f pg = f p ∘ρ(g).

Proof.

For v ∈ V,

$$\displaystyle{f_{pg}(v) = [pg,v] = [p,g \cdot v] = f_{p}(g \cdot v) = f_{p}\big(\rho (g)v\big).}$$

 □ 

Example 31.5.

Let π: P → M be a principal G-bundle. The vector bundle P × ρ V → M associated to the trivial representation \(\rho: G \rightarrow \mathop{\mathrm{GL}}\nolimits (V )\) is the trivial bundle M × V → M, for there is a vector bundle isomorphism

$$\displaystyle\begin{array}{rcl} & & \qquad \qquad \qquad \ \ \qquad P \times _{\rho }V \rightarrow M \times V, {}\\ & & [p,v] = [pg,g^{-1} \cdot v] = [pg,v]\mapsto \big(\pi (p),v\big), {}\\ \end{array}$$

with inverse map

$$\displaystyle{(x,v)\mapsto [p,v]\quad \text{for any }p \in \pi ^{-1}(x).}$$

In this case, for each p ∈ P the linear isomorphism f p : V → E x  = V, v ↦ [p, v], is the identity map.

31.3 Tensorial Forms on a Principal Bundle

We keep the same notation as in the previous section. Thus, π: P → M is a principal G-bundle, \(\rho: G \rightarrow \mathop{\mathrm{GL}}\nolimits (V )\) a finite-dimensional representation of G, and E: = P × ρ V the vector bundle associated to P via ρ.

Definition 31.6.

A V -valued k-form φ on P is said to be right-equivariant of type ρ or right-equivariant with respect to ρ if for every g ∈ G,

$$\displaystyle{r_{g}^{{\ast}}\varphi =\rho (g^{-1}) \cdot \varphi.}$$

What this means is that for p ∈ P and v 1, , v k  ∈ T p P,

$$\displaystyle{(r_{g}^{{\ast}}\varphi )_{ p}(v_{1},\ldots,v_{k}) =\rho (g^{-1})\big(\varphi _{ p}(v_{1},\ldots,v_{k})\big).}$$

In the literature (for example, [12, p. 75]), such a form is said to be pseudo-tensorial of type ρ.

Definition 31.7.

A V -valued k-form φ on P is said to be horizontal if φ vanishes whenever one of its arguments is a vertical vector. Since a 0-form never takes an argument, every 0-form on P is by definition horizontal.

Definition 31.8.

A V -valued k-form φ on P is tensorial of type ρ if it is right-equivariant of type ρ and horizontal. The set of all smooth tensorial V -valued k-forms of type ρ is denoted by Ω ρ k(P, V ).

Example.

Since the curvature Ω of a connection ω on a principal G-bundle P is horizontal and right-equivariant of type \(\mathop{\mathrm{Ad}}\nolimits\), it is tensorial of type \(\mathop{\mathrm{Ad}}\nolimits\).

The set Ω ρ k(P, V ) of tensorial k-forms of type ρ on P becomes a vector space with the usual addition and scalar multiplication of forms. These forms are of special interest because they can be viewed as forms on the base manifold M with values in the associated bundle E: = P × ρ V. To each tensorial V -valued k form φ ∈ Ω ρ k(P, V ) we associate a k-form φ  ∈ Ω k(M, E) as follows. Given x ∈ M and v 1, , v k  ∈ T x M, choose any point p in the fiber P x and choose lifts u 1, , u k at p of v 1, , v k , i.e., vectors in T p P such that π (u i ) = v i . Then φ is defined by

$$\displaystyle{ \varphi _{x}^{\flat }(v_{ 1},\ldots,v_{k}) = f_{p}\big(\varphi _{p}(u_{1},\ldots,u_{k})\big) \in E_{x}, }$$
(31.3)

where f p : V → E x is the isomorphism v ↦ [p, v] of the preceding section.

Conversely, if ψ ∈ Ω k(M, E), we define ψ  ∈ Ω ρ k(P, V ) as follows. Given p ∈ P and u 1, , u k  ∈ T p P, let x = π(p) and set

$$\displaystyle{ \psi _{p}^{\sharp }(u_{ 1},\ldots,u_{k}) = f_{p}^{-1}\big(\psi _{ x}(\pi _{{\ast}}u_{1},\ldots,\pi _{{\ast}}u_{k})\big) \in V. }$$
(31.4)

Theorem 31.9.

The map

$$\displaystyle\begin{array}{rcl} \varOmega _{\rho }^{k}(P,V )& \rightarrow & \varOmega ^{k}(M,E), {}\\ \varphi & \mapsto & \varphi ^{\flat }, {}\\ \end{array}$$

is a well-defined linear isomorphism with inverse \(\psi ^{\sharp } \leftarrowtail \psi\).

Proof.

To show that φ is well defined, we need to prove that the definition (31.3) is independent of the choice of p ∈ P x and of u 1, , u k  ∈ T p P. Suppose u1, , u k  ∈ T p P is another set of vectors such that π (u i ) = v i . Then π (u i u i ) = 0 so that u i u i is vertical. Since φ is horizontal and k-linear,

$$\displaystyle\begin{array}{rcl} \varphi _{p}(u^{\prime}_{1},\ldots,u^{\prime}_{k})& =& \varphi _{p}(u_{1} + \mathrm{vertical},\ldots,u_{k} + \mathrm{vertical}) {}\\ & =& \varphi _{p}(u_{1},\ldots,u_{k}). {}\\ \end{array}$$

This proves that for a given p ∈ P, the definition (31.3) is independent of the choice of lifts of v 1, , v k to p.

Next suppose we choose pg instead of p as the point in the fiber P x . Because πr g  = π,

$$\displaystyle{\pi _{{\ast}}(r_{g{\ast}}u_{i}) = (\pi \circ r_{g})_{{\ast}}u_{i} =\pi _{{\ast}}u_{i} = v_{i},}$$

so that r g u 1, , r g u k are lifts of v 1, , v k to pg. We have, by right equivariance with respect to ρ,

$$\displaystyle\begin{array}{rcl} \varphi _{pg}(r_{g{\ast}}u_{1},\ldots,r_{g{\ast}}u_{k})& =& (r_{g}^{{\ast}}\varphi _{ pg})(u_{1},\ldots,u_{k}) {}\\ & =& \rho (g^{-1})\varphi _{ p}(u_{1},\ldots,u_{k}). {}\\ \end{array}$$

So by Lemma 31.4,

$$\displaystyle\begin{array}{rcl} f_{pg}\big(\varphi _{pg}(r_{g{\ast}}u_{1},\ldots,r_{g{\ast}}u_{k})\big)& =& f_{pg}\big(\rho (g^{-1})\varphi _{ p}(u_{1},\ldots,u_{k})\big) {}\\ & =& \big(f_{p} \circ \rho (g)\big)\big(\rho (g^{-1})\varphi _{ p}(u_{1},\ldots,u_{k})\big) {}\\ & =& f_{p}\big(\varphi _{p}(u_{1},\ldots,u_{k})\big). {}\\ \end{array}$$

This proves that the definition (31.3) is independent of the choice of p in the fiber P x .

Let ψ ∈ Ω k(M, E). It is clear from the definition (31.4) that ψ is horizontal. It is easy to show that ψ is right-equivariant with respect to ρ (Problem 31.4). Hence, ψ  ∈ Ω ρ k(P, V ).

For v 1, , v k  ∈ T x M, choose p ∈ P x and vectors u 1, , u k  ∈ T p P that lift v 1, , v k . Then

$$\displaystyle\begin{array}{rcl} (\psi ^{\sharp \flat })_{ x}(v_{1},\ldots,v_{k})& =& f_{p}\big(\psi _{p}^{\sharp }(u_{ 1},\ldots,u_{k})\big) {}\\ & =& f_{p}\Big(f_{p}^{-1}\big(\psi _{ x}(\pi _{{\ast}}u_{1},\ldots,\pi _{{\ast}}u_{k})\big)\Big) {}\\ & =& \psi _{x}(v_{1},\ldots,v_{k}). {}\\ \end{array}$$

Hence, ψ ♯ ♭ = ψ.

Similarly, φ ♭ ♯ = φ for φ ∈ Ω ρ k(P, V ), which we leave to the reader to show (Problem 31.5). Therefore, the map ψψ is inverse to the map φφ . □ 

Example 31.10 (Curvature as a form on the base).

By Theorem 31.9, the curvature form Ω of a connection on a principal G-bundle P can be viewed as an element of \(\varOmega ^{2}(M,\mathop{\mathrm{Ad}}\nolimits P)\), a 2-form on M with values in the adjoint bundle \(\mathop{\mathrm{Ad}}\nolimits P\).

When k = 0 in Theorem 31.9, Ω p 0(P, V ) consists of maps f: P → V that are right-equivariant with respect to ρ:

$$\displaystyle{(r_{g}^{{\ast}}f)(p) =\rho (g)^{-1}f(p),}$$

or

$$\displaystyle{f(pg) =\rho (g^{-1})f(p) = g^{-1} \cdot f(p).}$$

On the right-hand side of Theorem 31.9,

$$\displaystyle{\varOmega ^{0}(M,P \times _{\rho }V ) =\varOmega ^{0}(M,E) = \text{ sections of the associated bundle E.}}$$

Hence, we have the following corollary.

Corollary 31.11.

Let G be a Lie group, P → M a principal G-bundle, and \(\rho: G \rightarrow \mathop{\mathrm{Aut}}\nolimits (V )\) a representation of G. There is a one-to-one correspondence

$$\displaystyle{\left \{G - equivariant\ mapsf: P \rightarrow V \right \}\quad \longleftrightarrow \quad \left \{sections\ of\ the\ associated\ bundle\ P \times _{\rho }V \rightarrow M\right \}.}$$

By the local triviality condition, for any principal bundle π: P → M the projection map π is a submersion and therefore the pullback map π : Ω (M) → Ω (P) is an injection. A differential form φ on P is said to be basic if it is the pullback π ψ of a form ψ on M; it is G-invariant if r g φ = φ for all g ∈ G. More generally, for any vector space V, these concepts apply to V -valued forms as well.

Suppose \(\rho: G \rightarrow \mathop{\mathrm{GL}}\nolimits (V )\) is the trivial representation \(\rho (g) =\mathbb{1}\) for all g ∈ G. Then an equivariant form φ of type ρ on P satisfies

$$\displaystyle{r_{g}^{{\ast}}\varphi =\rho (g^{-1})\cdot \varphi =\varphi \quad \mbox{ for all $g \in G$}.}$$

Thus, an equivariant form of type ρ for the trivial representation ρ is exactly an invariant form on P. Unravelling Theorem 31.9 for a trivial representation will give the following theorem.

Theorem 31.12.

Let π: P → M be a principal G-bundle and V a vector space. A V-valued form on P is basic if and only if it is horizontal and G-invariant.

Proof.

Let \(\rho: G \rightarrow \mathop{\mathrm{GL}}\nolimits (V )\) be the trivial representation. As noted above, Ω ρ k(P, V ) consists of horizontal, G-invariant V -valued k-forms on P.

By Example 31.5, when ρ is the trivial representation, the vector bundle E = P × ρ V is the product bundle M × V over M and for each p ∈ P, the linear isomorphism f p : V → E x  = V, where x = π(p), is the identity map. Then the isomorphism

$$\displaystyle\begin{array}{rcl} \varOmega ^{k}(M,E) =\varOmega ^{k}(M,M \times V ) =\varOmega ^{k}(M,V )& \rightarrow & \varOmega _{\rho }^{k}(P,V ), {}\\ \psi & \mapsto & \psi ^{\#}, {}\\ \end{array}$$

is given by

$$\displaystyle\begin{array}{rcl} \psi _{p}^{\#}(u_{ 1},\ldots,u_{k})& =& \psi _{x}(\pi _{{\ast}}u_{1},\ldots,\pi _{{\ast}}u_{k})\quad \mathrm{(by\ (31.4))} {}\\ & =& (\pi ^{{\ast}}\psi )_{ p}(u_{1},\ldots,u_{k}). {}\\ \end{array}$$

Therefore,

$$\displaystyle{\psi ^{\#} =\pi ^{{\ast}}\psi.}$$

This proves that horizontal, G-invariant forms on P are precisely the basic forms. □ 

31.4 Covariant Derivative

Recall that the existence of a connection ω on a principal G-bundle π: P → M is equivalent to the decomposition of the tangent bundle TP into a direct sum of the vertical subbundle \(\mathcal{V}\) and a smooth right-invariant horizontal subbundle \(\mathcal{H}\). For any vector X p  ∈ T p P, we write

$$\displaystyle{X_{p} = vX_{p} + hX_{p}}$$

as the sum of its vertical and horizontal components. This will allow us to define a covariant derivative of vector-valued forms on P. By the isomorphism of Theorem 31.9, we obtain in turn a covariant derivative of forms on M with values in an associated bundle.

Let \(\rho: G \rightarrow \mathop{\mathrm{GL}}\nolimits (V )\) be a finite-dimensional representation of G and let E: = P × ρ V be the associated vector bundle.

Proposition 31.13.

If φ ∈Ω k (P,V ) is right-equivariant of type ρ, then so is dφ.

Proof.

For a fixed g ∈ G,

$$\displaystyle\begin{array}{rcl} r_{g}^{{\ast}}d\varphi & =& dr_{ g}^{{\ast}}\varphi = d\rho (g^{-1})\varphi {}\\ & =& \rho (g^{-1})d\varphi, {}\\ \end{array}$$

since ρ(g −1) is a constant linear map for a fixed g. □ 

In general, the exterior derivative does not preserve horizontality. For any V -valued k-form φ on P, we define its horizontal component φ h ∈ Ω k(P, V ) as follows: for p ∈ P and v 1, , v k  ∈ T p P,

$$\displaystyle{\varphi _{p}^{h}(v_{ 1},\ldots,v_{k}) =\varphi _{p}(hv_{1},\ldots,hv_{k}).}$$

Proposition 31.14.

If φ ∈Ω k (P,V ) is right-equivariant of type ρ, then so is φ h.

Proof.

For g ∈ G, p ∈ P, and v 1, , v k  ∈ T p P,

$$\displaystyle\begin{array}{rcl} r_{g}^{{\ast}}(\varphi _{ pg}^{h})(v_{ 1},\ldots,v_{k})& =& \varphi _{pg}^{h}(r_{ g{\ast}}v_{1},\ldots,r_{g{\ast}}v_{k})\quad \qquad \text{(definition of pullback)} {}\\ & =& \varphi _{pg}(hr_{g{\ast}}v_{1},\ldots,hr_{g{\ast}}v_{k})\qquad \mbox{(definition of $\varphi ^{h}$)} {}\\ & =& \varphi _{pg}(r_{g{\ast}}hv_{1},\ldots,r_{g{\ast}}hv_{k})\qquad \mathrm{(Proposition\ 28.4)} {}\\ & =& (r_{g}^{{\ast}}\varphi _{ pg})(hv_{1},\ldots,hv_{k}) {}\\ & =& \rho (g^{-1}) \cdot \varphi _{ p}(hv_{1},\ldots,hv_{k})\quad \ \mbox{(right-equivariance of $\varphi$ )} {}\\ & =& \rho (g^{-1}) \cdot \varphi _{ p}^{h}(v_{ 1},\ldots,v_{k}) {}\\ \end{array}$$

 □ 

Propositions 31.13 and 31.14 together imply that if φ ∈ Ω k(P, V ) is right-equivariant of type ρ, then (d φ)h ∈ Ω k+1(P, V ) is horizontal and right-equivariant of type ρ, i.e., tensorial of type ρ.

Definition 31.15.

Let π: P → M be a principal G-bundle with a connection ω and let V be a real vector space. The covariant derivative of a V -valued k-form φ ∈ Ω k(P, V ) is D φ = (d φ)h.

Let \(\rho: G \rightarrow \mathop{\mathrm{GL}}\nolimits (V )\) be a finite-dimensional representation of the Lie group G. The covariant derivative is defined for any V -valued k-form on P, and it maps a right-equivariant form of type ρ to a tensorial form of type ρ. In particular, it restricts to a map

$$\displaystyle{ D: \varOmega _{\rho }^{k}(P,V ) \rightarrow \varOmega _{\rho }^{k+1}(P,V ) }$$
(31.5)

on the space of tensorial forms.

Proposition 31.16.

Let π: P → M be a principal G-bundle with a connection and \(\rho: G \rightarrow \mathop{\mathrm{GL}}\nolimits (V )\) a representation of G. The covariant derivative

$$\displaystyle{D: \varOmega _{\rho }^{k}(P,V ) \rightarrow \varOmega _{\rho }^{k+1}(P,V )}$$

on tensorial forms of type ρ is an antiderivation of degree + 1.

Proof.

Let ω, τ ∈ Ω ρ (P, V ) be tensorial forms of type ρ. Then

$$\displaystyle\begin{array}{rcl} D(\omega \wedge \tau )& =& \big(d(\omega \wedge \tau )\big)^{h} {}\\ & =& \big((d\omega ) \wedge \tau +(-1)^{\deg \omega }\omega \wedge d\tau )^{h} {}\\ & =& (d\omega )^{h} \wedge \tau ^{h} + (-1)^{\deg \omega }\omega ^{h} \wedge (d\tau )^{h} {}\\ & =& Dw \wedge \tau ^{h} + (-1)^{\deg \omega }\omega ^{h} \wedge D\tau. {}\\ \end{array}$$

Since τ and ω are horizontal, τ h = τ and ω h = ω. Therefore,

$$\displaystyle{D(\omega \wedge \tau ) = D\omega \wedge \tau +(-1)^{\deg \omega }\omega \wedge D\tau.}$$

⊓⊔

If E: = P × ρ V is the associated vector bundle via the representation ρ, then the isomorphism of Theorem 31.9 transforms the linear map (31.5) into a linear map

$$\displaystyle{D: \varOmega ^{k}(M,E) \rightarrow \varOmega ^{k+1}(M,E).}$$

Unlike the exterior derivative, the covariant derivative depends on the choice of a connection on P. Moreover, D 2 ≠ 0 in general.

Example 31.17 (Curvature of a principal bundle).

By Theorem 30.4 the curvature form \(\varOmega \in \varOmega _{\mathop{\mathrm{Ad}}\nolimits }^{2}(P,\mathfrak{g})\) on a principal bundle is the covariant derivative D ω of the connection form \(\omega \in \varOmega ^{1}(P,\mathfrak{g})\). Because ω is not horizontal, it is not in \(\varOmega _{\mathop{\mathrm{Ad}}\nolimits }^{1}(P,\mathfrak{g})\).

31.5 A Formula for the Covariant Derivative of a Tensorial Form

Let π: P → M be a smooth principal G-bundle with a connection ω, and let \(\rho: G \rightarrow \mathop{\mathrm{GL}}\nolimits (V )\) be a finite-dimensional representation of G. In the preceding section we defined the covariant derivative of a V -valued k-form φ on P: D φ = (d φ)h, the horizontal component of d φ. In this section we derive a useful alternative formula for the covariant derivative, but only for a tensorial form.

The Lie group representation \(\rho: G \rightarrow \mathop{\mathrm{GL}}\nolimits (V )\) induces a Lie algebra representation \(\rho _{{\ast}}: \mathfrak{g} \rightarrow \mathfrak{g}\mathfrak{l}(V )\), which allows us to define a product of a \(\mathfrak{g}\)-valued k-form τ and a V -valued -form φ on P: for p ∈ P and v 1, , v k+  ∈ T p P,

$$\displaystyle\begin{array}{rcl} & & (\tau \cdot \varphi )_{p}(v_{1},\ldots,v_{k+\ell}) {}\\ & & \phantom{(\tau \cdot \varphi )_{p}1} = \frac{1} {k!\ell!}\sum _{\sigma \in S_{k+\ell}}\mathop{ \mathrm{sgn}}\nolimits (\sigma )\rho _{{\ast}}\big(\tau _{p}(v_{\sigma (1)},\ldots,v_{\sigma (k)})\big)\varphi _{p}\left (v_{\sigma (k+1)},\ldots,v_{\sigma (k+\ell)}\right ). {}\\ \end{array}$$

For the same reason as the wedge product, τ ⋅ φ is multilinear and alternating in its arguments; it is therefore a (k + )-covector with values in V.

Example 31.18.

If \(V = \mathfrak{g}\) and \(\rho =\mathop{ \mathrm{Ad}}\nolimits: G \rightarrow \mathop{\mathrm{GL}}\nolimits (\mathfrak{g})\) is the adjoint representation, then

$$\displaystyle{(\tau \cdot \varphi )_{p} = \frac{1} {k!\ell!}\sum _{\sigma \in S_{k+\ell}}\mathop{ \mathrm{sgn}}\nolimits (\sigma )\left [\tau _{p}(v_{\sigma (1)},\ldots,v_{\sigma (k)}),\varphi _{p}(v_{\sigma (k+1)},\ldots,v_{\sigma (k+\ell)})\right ].}$$

In this case we also write [τ, φ] instead of τ ⋅ φ.

Theorem 31.19.

Let π: P → M be a principal G-bundle with connection form ω, and \(\rho: G \rightarrow \mathop{\mathrm{GL}}\nolimits (V )\) a finite-dimensional representation of G. If φ ∈Ω ρ k (P,V ) is a V -valued tensorial form of type ρ, then its covariant derivative is given by

$$\displaystyle{D\varphi = d\varphi +\omega \cdot \varphi.}$$

Proof.

Fix p ∈ P and v 1, , v k+1 ∈ T p P. We need to show that

$$\displaystyle\begin{array}{rcl} & & (d\varphi )_{p}(hv_{1},\ldots,hv_{k+1}) = (d\varphi )_{p}(v_{1},\ldots,v_{k+1}) \\ & & \phantom{(d\varphi )_{p}(hv_{1},\ldots.)} + \frac{1} {k!}\sum _{\sigma \in S_{k+1}}\mathop{ \mathrm{sgn}}\nolimits (\sigma )\rho _{{\ast}}\left (\omega _{p}(v_{\sigma (1)})\right )\varphi _{p}\left (v_{\sigma (2)},\ldots,v_{\sigma (k+1)}\right ).{}\end{array}$$
(31.6)

Because both sides of (31.6) are linear in each argument v i , which may be decomposed into the sum of a vertical and a horizontal component, we may assume that each v i is either vertical or horizontal. By Lemma 30.3, throughout the proof we may further assume that the vectors v 1, , v k+1 have been extended to vector fields X 1, , X k+1 on P each of which is either vertical or horizontal. If X i is vertical, then it is a fundamental vector field A i for some \(A_{i} \in \mathfrak{g}\). If X i is horizontal, then it is the horizontal lift \(\tilde{B_{i}}\) of a vector field B i on M. By construction, \(\tilde{B_{i}}\) is right-invariant (Proposition 28.6).

Instead of proving (31.6) at a point p, we will prove the equality of functions

$$\displaystyle{ (d\varphi )(hX_{1},\ldots,hX_{k+1}) = \mathrm{I} + \mathrm{II}, }$$
(31.7)

where

$$\displaystyle\begin{array}{rcl} \mathrm{I}& =& (d\varphi )(X_{1},\ldots,X_{k+1}) {}\\ \end{array}$$

and

$$\displaystyle\begin{array}{rcl} \mathrm{II} = \frac{1} {k!}\sum _{\sigma \in S_{k+1}}\mathop{ \mathrm{sgn}}\nolimits (\sigma )\rho _{{\ast}}\left (\omega (X_{\sigma (1)})\right )\varphi \left (X_{\sigma (2)},\ldots,X_{\sigma (k+1)}\right ).& & {}\\ \end{array}$$

Case 1. The vector fields X 1 ,…,X k+1 are all horizontal.Then II = 0 because ω(X σ(1)) = 0 for all σ ∈ S k+1. In this case, (31.7) is trivially true.

Case 2. At least two of X 1 ,…,X k+1 are vertical.By the skew-symmetry of the arguments, we may assume that X 1 = A 1 and X 2 = A 2 are vertical. By Problem 27.1, [X 1, X 2] = [A 1, A 2] is also vertical.

The left-hand side of (31.7) is zero because hX 1 = 0. By the global formula for the exterior derivative [21, Th. 20.14, p. 233],

$$\displaystyle{\mathrm{I} =\sum _{ i=1}^{k+1}(-1)^{i-1}X_{ i}\varphi (\ldots,\widehat{X_{i}},\ldots ) +\sum _{1\leq i<j\leq k+1}(-1)^{i+j}\varphi ([X_{ i},X_{j}],\ldots,\widehat{X_{i}},\ldots,\widehat{X_{j}},\ldots ).}$$

In this expression every term in the first sum is zero because φ is horizontal and at least one of its arguments is vertical. In the second sum at least one of the arguments of φ is X 1, X 2, or [X 1, X 2], all of which are vertical. Therefore, every term in the second sum in I is also zero.

As for II in (31.7), in every term at least one of the arguments of φ is vertical, so II = 0.

Case 3. The first vector field X 1 = A is vertical; the rest X2,…,Xk+1 are horizontal and right-invariant.The left-hand side of (31.7) is clearly zero because hX 1 = 0.

On the right-hand side,

$$\displaystyle\begin{array}{rcl} \mathrm{I}& =& (d\varphi )(X_{1},\ldots,X_{k+1}) {}\\ & =& \sum (-1)^{i+1}X_{ i}\varphi (X_{1},\ldots,\widehat{X_{i}},\ldots,X_{k+1}) {}\\ & & \phantom{\sum (-1)^{i+1}X_{ i}\varphi } +\sum (-1)^{i+j}\varphi ([X_{ i},X_{j}],X_{1},\ldots,\widehat{X_{i}},\ldots,\widehat{X_{j}},\ldots,X_{k+1}). {}\\ \end{array}$$

Because φ is horizontal and X 1 is vertical, the only nonzero term in the first sum is

$$\displaystyle{X_{1}\varphi (X_{2},\ldots,X_{k+1}) =\underline{ A}\varphi (X_{2},\ldots,X_{k+1})}$$

and the only nonzero terms in the second sum are

$$\displaystyle{\sum _{j=2}^{k+1}(-1)^{1+j}\varphi ([X_{ 1},X_{j}],\widehat{X}_{1},X_{2},\ldots,\widehat{X}_{j},\ldots,X_{k+1}).}$$

Since the X j , j = 2, , k + 1, are right-invariant horizontal vector fields, by Lemma 28.7,

$$\displaystyle{[X_{1},X_{j}] = [\underline{A},X_{j}] = 0.}$$

Therefore,

$$\displaystyle{\mathrm{I} =\underline{ A}\varphi (X_{2},\ldots,X_{k+1}).}$$

If σ(i) = 1 for any i ≥ 2, then

$$\displaystyle{\varphi (X_{\sigma (2)},\ldots,X_{\sigma (k+1)}) = 0.}$$

It follows that the nonzero terms in II all satisfy σ(1) = 1 and

$$\displaystyle\begin{array}{rcl} \mathrm{II}& =& \frac{1} {k!}\sum _{\begin{array}{c}\sigma \in S_{k+1} \\ \sigma (1)=1 \end{array}}\mathop{ \mathrm{sgn}}\nolimits (\sigma )\rho _{{\ast}}\big(\omega (X_{1})\big)\varphi \left (X_{\sigma (2)},\ldots,X_{\sigma (k+1)}\right ) {}\\ & =& \frac{1} {k!}\sum _{\begin{array}{c}\sigma \in S_{k+1} \\ \sigma (1)=1 \end{array}}\mathop{ \mathrm{sgn}}\nolimits (\sigma )\rho _{{\ast}}(A)\varphi \left (X_{\sigma (2)},\ldots,X_{\sigma (k+1)}\right ) {}\\ & =& \rho _{{\ast}}(A)\varphi \left (X_{2},\ldots,X_{k+1}\right )\qquad \mbox{ (because $\varphi $ is alternating)}. {}\\ \end{array}$$

Denote by f the function φ(X 2, , X k+1) on P. For p ∈ P, to calculate A p f, choose a curve c(t) in G with initial point c(0) = e and initial vector c′(0) = A, for example, c(t) = exp(tA). Then with j p : G → P being the map j p (g) = p ⋅ g,

$$\displaystyle\begin{array}{rcl} \underline{A}_{p}f& =& j_{p{\ast}}(A)f = j_{p{\ast}}\big(c^{\prime}(0)\big)f = j_{p{\ast}}\left (c_{{\ast}}\left (\left. \frac{d} {dt}\right \vert _{t=0}\right )\right )f {}\\ & =& (j_{p} \circ c)_{{\ast}}\left (\left. \frac{d} {dt}\right \vert _{t=0}\right )f = \left. \frac{d} {dt}\right \vert _{t=0}(f \circ j_{p} \circ c). {}\\ \end{array}$$

By the right-invariance of the horizontal vector fields X 2, , X k+1,

$$\displaystyle\begin{array}{rcl} (f \circ j_{p} \circ c)(t)& =& f\big(pc(t)\big) {}\\ & =& \varphi _{pc(t)}\left (X_{2,pc(t)},\ldots,X_{k+1,pc(t)}\right ) {}\\ & =& \varphi _{pc(t)}\left (r_{c(t){\ast}}X_{2,p},\ldots,r_{c(t){\ast}}X_{k+1,p}\right ) {}\\ & =& r_{c(t)}^{{\ast}}\varphi _{ pc(t)}\left (X_{2,p},\ldots,X_{k+1,p}\right ) {}\\ & =& \rho (c(t)^{-1})\varphi _{ p}\left (X_{2,p},\ldots,X_{k+1,p}\right )\qquad \qquad \mbox{ (right-equivariance of $\varphi $)} {}\\ & =& \rho (c(t)^{-1})f(p). {}\\ \end{array}$$

Differentiating this expression with respect to t and using the fact that the differential of the inverse is the negative [21, Problem 8.8(b)], we have

$$\displaystyle{\underline{A}_{p}f = (f \circ j_{p} \circ c)^{\prime}(0) = -\rho _{{\ast}}(c^{\prime}(0))f(p) = -\rho _{{\ast}}(A)f(p).}$$

So the right-hand side of (31.7) is

$$\displaystyle{\mathrm{I} + \mathrm{II} =\underline{ A}f +\rho _{{\ast}}(A)f = -\rho _{{\ast}}(A)f +\rho _{{\ast}}(A)f = 0.}$$

 □ 

If V is the Lie algebra \(\mathfrak{g}\) of a Lie group G and ρ is the adjoint representation of G, then ω ⋅ φ = [ω, φ]. In this case, for any tensorial k-form \(\varphi \in \varOmega _{\mathop{\mathrm{Ad}}\nolimits }^{k}(P,\mathfrak{g})\),

$$\displaystyle{D\varphi = d\varphi + [\omega,\varphi ].}$$

Although the covariant derivative is defined for any V -valued form on P, Theorem 31.19 is true only for tensorial forms. Since the connection form ω is not tensorial, Theorem 31.19 cannot be applied to ω. In fact, by the definition of the curvature form,

$$\displaystyle{\varOmega = d\omega + \frac{1} {2}[\omega,\omega ].}$$

By Theorem 30.4, Ω = (d ω)h = D ω. Combining these two expressions for the curvature, one obtains

$$\displaystyle{D\omega = d\omega + \frac{1} {2}[\omega,\omega ].}$$

The factor of 1∕2 shows that Theorem 31.19 is not true when applied to ω.

Since the curvature form Ω on a principal bundle P is tensorial of type \(\mathop{\mathrm{Ad}}\nolimits\), Theorem 31.19 applies and the second Bianchi identity (Theorem 30.4) may be restated as

$$\displaystyle{ D\varOmega = d\varOmega + [\omega,\varOmega ] = 0. }$$
(31.8)

Problems

Unless otherwise specified, in the following problems G is a Lie group with Lie algebra \(\mathfrak{g}\), π: P → M a principal G-bundle, \(\rho: G \rightarrow \mathop{\mathrm{GL}}\nolimits (V )\) a finite-dimensional representation of G, and E = P × ρ V the associated bundle.

31.1. Transition functions of an associated bundle

Show that if {(U α , ϕ α )} is a trivialization for P with transition functions g α β : U α U β  → G, then there is a trivialization {(U α , ψ α )} for E with transition functions \(\rho \circ g_{\alpha \beta }: U_{\alpha } \cap U_{\beta } \rightarrow \mathop{\mathrm{GL}}\nolimits (V )\).

31.2. Vector bundle structure on an associated bundle

Show that the operations (31.2) on E = P × ρ V are well defined and make the associated bundle β: E → M into a vector bundle.

31.3. Associated bundle of a frame bundle

Let E → M be a vector bundle of rank r and \(\mathop{\mathrm{Fr}}\nolimits (E) \rightarrow M\) its frame bundle. Show that the vector bundle associated to \(\mathop{\mathrm{Fr}}\nolimits (E)\) via the identity representation \(\rho: \mathop{ \mathrm{GL}}\nolimits (r, \mathbb{R}) \rightarrow \mathop{\mathrm{GL}}\nolimits (r, \mathbb{R})\) is isomorphic to E.

31.4. Tensorial forms

Prove that if ψ ∈ Ω k(M, P × ρ V ), then ψ  ∈ Ω k(P, V ) is right-equivariant with respect to ρ.

31.5. Tensorial forms

For φ ∈ Ω ρ k(P, V ), prove that φ ♭ ♯ = φ.

§32 Characteristic Classes of Principal Bundles

To a real vector bundle E → M of rank r, one can associate its frame bundle Fr(E) → M, a principal \(\mathop{\mathrm{GL}}\nolimits (r, \mathbb{R})\)-bundle. Similarly, to a complex vector bundle of rank r, one can associate its frame bundle, a principal \(\mathop{\mathrm{GL}}\nolimits (r, \mathbb{C})\)-bundle and to an oriented real vector bundle of rank r, one can associate its oriented frame bundle, a principal \(\mathop{\mathrm{GL}}\nolimits ^{+}(r, \mathbb{R})\)-bundle, where \(\mathop{\mathrm{GL}}\nolimits ^{+}(r, \mathbb{R})\) is the group of all r × r matrices of positive determinant. The Pontrjagin classes of a real vector bundle, the Chern classes of a complex vector bundle, and the Euler class of an oriented real vector bundle may be viewed as characteristic classes of the associated principal G-bundle for \(G =\mathop{ \mathrm{GL}}\nolimits (r, \mathbb{R}),\mathop{\mathrm{GL}}\nolimits (r, \mathbb{C})\), and \(\mathop{\mathrm{GL}}\nolimits ^{+}(r, \mathbb{R})\), respectively.

In this section we will generalize the construction of characteristic classes to principal G-bundles for any Lie group G. These are some of the most important diffeomorphism invariants of a principal bundle.

32.1 Invariant Polynomials on a Lie Algebra

Let V be a vector space of dimension n and V its dual space. An element of \(\mathop{\mathrm{Sym}}\nolimits ^{k}(V ^{\vee })\) is called a polynomial of degree k on V. Relative to a basis e 1, , e n for V and corresponding dual basis α 1, , α n for V , a function \(f: V \rightarrow \mathbb{R}\) is a polynomial of degree k if and only if it is expressible as a sum of monomials of degree k in α 1, , α n:

$$\displaystyle{ f =\sum a_{I}\alpha ^{i_{1} }\cdots \alpha ^{i_{k} }. }$$
(32.1)

For example, if \(V = \mathbb{R}^{n\times n}\) is the vector space of all n × n matrices, then \(\mathop{\mathrm{tr}}\nolimits X\) is a polynomial of degree 1 on V and detX is a polynomial of degree n on V.

Suppose now that \(\mathfrak{g}\) is the Lie algebra of a Lie group G. A polynomial \(f: \mathfrak{g} \rightarrow \mathbb{R}\) is said to be \(\mathop{\mathrm{Ad}}\nolimits (G)\)-invariant if for all g ∈ G and \(X \in \mathfrak{g}\),

$$\displaystyle{f\big((\mathop{\mathrm{Ad}}\nolimits g)X\big) = f(X).}$$

For example, if G is the general linear group \(\mathop{\mathrm{GL}}\nolimits (n, \mathbb{R})\), then \((\mathop{\mathrm{Ad}}\nolimits g)X = gXg^{-1}\) and \(\mathop{\mathrm{tr}}\nolimits X\) and detX are \(\mathop{\mathrm{Ad}}\nolimits G\)-invariant polynomials on the Lie algebra \(\mathfrak{g}\mathfrak{l}(n, \mathbb{R})\).

32.2 The Chern–Weil Homomorphism

Let G be a Lie group with Lie algebra \(\mathfrak{g}\), P → M a principal G-bundle, ω an Ehresmann connection on P, and Ω the curvature form of ω. Fix a basis e 1, , e n for \(\mathfrak{g}\) and dual basis α 1, , α n for \(\mathfrak{g}^{\vee }\). Then the curvature form Ω is a linear combination

$$\displaystyle{\varOmega =\sum \varOmega ^{i}e_{ i},}$$

where the coefficients Ω i are real-valued 2-forms on P. If \(f: \mathfrak{g} \rightarrow \mathbb{R}\) is the polynomial \(\sum a_{I}\alpha ^{i_{1}}\cdots \alpha ^{i_{k}}\), we define f(Ω) to be the 2k-form

$$\displaystyle{f(\varOmega ) =\sum a_{I}\varOmega ^{i_{1} } \wedge \cdots \wedge \varOmega ^{i_{k} }}$$

on P. Although defined in terms of a basis for \(\mathfrak{g}\), the 2k-form f(Ω) is independent of the choice of a basis (Problem 32.2).

Recall that the covariant derivative D φ of a k-form φ on a principal bundle P is given by

$$\displaystyle{(D\varphi )_{p}(v_{1},\ldots,v_{k}) = (d\varphi )_{p}(hv_{1},\ldots,hv_{k}),}$$

where v i  ∈ T p P and hv i is the horizontal component of v i .

Lemma 32.1.

Let π: P → M be a principal bundle. If φ is a basic form on P, then dφ = Dφ.

Proof.

A tangent vector X p  ∈ T p P decomposes into the sum of its vertical and horizontal components:

$$\displaystyle{X_{p} = vX_{p} + hX_{p}.}$$

Here h: T p P → T p P is the map that takes a tangent vector to its horizontal component. Since π X p  = π hX p for all X p  ∈ T p P, we have

$$\displaystyle{\pi _{{\ast}} =\pi _{{\ast}}\circ h.}$$

Suppose φ = π τ for τ ∈ Ω k(M). Then

$$\displaystyle\begin{array}{rcl} D\varphi & =& (d\varphi ) \circ h\quad\quad\mbox{ (definition of $D$)} {}\\ & =& (d\pi ^{{\ast}}\tau ) \circ h\quad \mbox{ ($\varphi $ is basic)} {}\\ & =& (\pi ^{{\ast}}d\tau ) \circ h\quad\mbox{ ([21, Prop. 19.5])} {}\\ & =& d\tau \circ \pi _{{\ast}}\circ h\quad\mbox{ (definition of $\pi ^{{\ast}}$)} {}\\ & =& d\tau \circ \pi _{{\ast}}\qquad\ \mbox{ ($\pi _{{\ast}}\circ h =\pi _{{\ast}}$)} {}\\ & =& \pi ^{{\ast}}d\tau \qquad\ \quad\mbox{ (definition of $\pi ^{{\ast}}$)} {}\\ & =& d\pi ^{{\ast}}\tau \qquad\ \quad\mbox{ ([21, Prop. 19.5])} {}\\ & =& d\varphi (\varphi =\pi ^{{\ast}}\tau ). {}\\ \end{array}$$

⊓⊔

The Chern–Weil homomorphism is based on the following theorem. As before, G is a Lie group with Lie algebra \(\mathfrak{g}\).

Theorem 32.2.

Let Ω be the curvature of a connection ω on a principal G-bundle π: P → M, and f an \(\mathop{\mathrm{Ad}}\nolimits (G)\) -invariant polynomial of degree k on \(\mathfrak{g}\) . Then

(i):

f(Ω) is a basic form on P, i.e., there exists a 2k-form Λ on M such that f(Ω) = π Λ.

(ii):

Λ is a closed form.

(iii):

The cohomology class [Λ] is independent of the connection.

Proof.

(i):

Since the curvature Ω is horizontal, so are its components Ω i and therefore so is \(f(\varOmega ) =\sum a_{I}\varOmega ^{i_{1}} \wedge \cdots \wedge \varOmega ^{i_{k}}.\)

To check the G-invariance of f(Ω), let g ∈ G. Then

$$\displaystyle\begin{array}{rcl} r_{g}^{{\ast}}\big(f(\varOmega )\big)& =& r_{ g}^{{\ast}}\big(\sum a_{ I}\varOmega ^{i_{1} } \wedge \cdots \wedge \varOmega ^{i_{k} }\big) {}\\ & =& \sum a_{I}r_{g}^{{\ast}}(\varOmega ^{i_{1} }) \wedge \cdots \wedge r_{g}^{{\ast}}(\varOmega ^{i_{k} }). {}\\ \end{array}$$

Since the curvature form Ω is right-equivariant,

$$\displaystyle{r_{g}^{{\ast}}\varOmega = (\mathop{\mathrm{Ad}}\nolimits g^{-1})\varOmega }$$

or

$$\displaystyle{r_{g}^{{\ast}}(\sum \varOmega ^{i}e_{ i}) =\sum \big ((\mathop{\mathrm{Ad}}\nolimits g^{-1})\varOmega \big)^{i}e_{ i},}$$

so that

$$\displaystyle{r_{g}^{{\ast}}(\varOmega ^{i}) =\big ((\mathop{\mathrm{Ad}}\nolimits g^{-1})\varOmega \big)^{i}.}$$

Thus,

$$\displaystyle\begin{array}{rcl} r_{g}^{{\ast}}\big(f(\varOmega )\big)& =& \sum a_{ I}\big((\mathop{\mathrm{Ad}}\nolimits g^{-1})\varOmega \big)^{i_{1} } \wedge \cdots \wedge \big ((\mathop{\mathrm{Ad}}\nolimits g^{-1})\varOmega \big)^{i_{k} } {}\\ & =& f\big((\mathop{\mathrm{Ad}}\nolimits g^{-1})\varOmega \big) {}\\ & =& f(\varOmega )\qquad \mbox{ (by the $\mathop{\mathrm{Ad}}\nolimits G$-invariance of $f$)}. {}\\ \end{array}$$

Since f(Ω) is horizontal and G-invariant, by Theorem 31.12, it is basic.

(ii):

Since π : T p P → T π(p) M is surjective, π : Ω (M) → Ω (P) is injective. Therefore, to show that d Λ = 0, it suffices to show that

$$\displaystyle{\pi ^{{\ast}}d\varLambda = d\pi ^{{\ast}}\varLambda = df(\varOmega ) = 0.}$$

If \(f =\sum a_{I}\alpha ^{i_{1}}\cdots \alpha ^{i_{k}}\), then

$$\displaystyle{f(\varOmega ) =\sum a_{I}\varOmega ^{i_{1} } \wedge \cdots \wedge \varOmega ^{i_{k} }.}$$

In this expression, each a I is a constant and therefore by Lemma 32.1

$$\displaystyle{Da_{I} = da_{I} = 0.}$$

By the second Bianchi identity (31.8), D Ω = 0. Therefore, D Ω i = 0 for each i. Since the Ω i are right-equivariant of type \(\mathop{\mathrm{Ad}}\nolimits\) and horizontal, they are tensorial forms. By Lemma 32.1 and because D is an antiderivation on tensorial forms (Proposition 31.16)

$$\displaystyle\begin{array}{rcl} d\big(f(\varOmega )\big)& =& D\big(f(\varOmega )\big) = D\big(\sum a_{I}\varOmega ^{i_{1} } \wedge \cdots \wedge \varOmega ^{i_{k} }\big) {}\\ & =& \sum _{I}\sum _{j}a_{I}\varOmega ^{i_{1} } \wedge \cdots \wedge D\varOmega ^{i_{j} } \wedge \cdots \wedge \varOmega ^{i_{2k} } {}\\ & =& 0. {}\\ \end{array}$$
(iii):

Let I be an open interval containing the closed interval [0, 1]. Then P × I is a principal G-bundle over M × I. Denote by ρ the projection P × I → P to the first factor. If ω 0 and ω 1 are two connections on P, then

$$\displaystyle{ \tilde{\omega }= (1 - t)\rho ^{{\ast}}\omega _{ 0} + t\rho ^{{\ast}}\omega _{ 1} }$$
(32.2)

is a connection on P × I (Check the details). Moreover, if i t : P → P × I is the inclusion p ↦ (p, t), then \(i_{0}^{{\ast}}\tilde{\omega } =\omega _{0}\) and \(i_{1}^{{\ast}}\tilde{\omega } =\omega _{1}\).

Let

$$\displaystyle{\tilde{\varOmega }= d\tilde{\omega } + \frac{1} {2}[\tilde{\omega },\tilde{\omega }]}$$

be the curvature of the connection \(\tilde{\omega }\). It pulls back under i 0 to

$$\displaystyle\begin{array}{rcl} i_{0}^{{\ast}}\tilde{\varOmega }& =& d\imath _{ 0}^{{\ast}}\tilde{\omega } + \frac{1} {2}i_{0}^{{\ast}}[\tilde{\omega },\tilde{\omega }] {}\\ & =& d\omega _{0} + \frac{1} {2}[i_{0}^{{\ast}}\tilde{\omega },i_{ 0}^{{\ast}}\tilde{\omega }] {}\\ & =& d\omega _{0} + \frac{1} {2}[\omega _{0},\omega _{0}] {}\\ & =& \varOmega _{0}, {}\\ \end{array}$$

the curvature of the connection ω 0. Similarly, \(i_{1}^{{\ast}}\tilde{\varOmega } =\varOmega _{1}\), the curvature of the connection ω 1.

For any \(\mathop{\mathrm{Ad}}\nolimits (G)\)-invariant polynomial

$$\displaystyle{f =\sum a_{I}\alpha ^{i_{1} }\cdots \alpha ^{i_{k} }}$$

of degree k on \(\mathfrak{g}\),

$$\displaystyle\begin{array}{rcl} i_{0}^{{\ast}}f(\tilde{\varOmega })& =& i_{ 0}^{{\ast}}\sum a_{ I}\tilde{\varOmega }^{i_{1} } \wedge \cdots \wedge \tilde{\varOmega }^{i_{k} } {}\\ & =& \sum a_{I}\varOmega _{0}^{i_{1} } \wedge \cdots \wedge \varOmega _{0}^{i_{k} } {}\\ & =& f(\varOmega _{0}) {}\\ \end{array}$$

and

$$\displaystyle{i_{1}^{{\ast}}f(\tilde{\varOmega }) = f(\varOmega _{ 1}).}$$

Note that i 0 and i 1: P → P × I are homotopic through the homotopy i t . By the homotopy axiom of de Rham cohomology, the cohomology classes \([i_{0}^{{\ast}}f(\tilde{\varOmega })]\) and \([i_{1}^{{\ast}}f(\tilde{\varOmega })]\) are equal. Thus, [f(Ω 0)] = [f(Ω 1)], or

$$\displaystyle{\pi ^{{\ast}}[\varLambda _{ 0}] =\pi ^{{\ast}}[\varLambda _{ 1}].}$$

By the injectivity of π , [Λ 0] = [Λ 1], so the cohomology class of Λ is independent of the connection. □ 

Let π: P → M be a principal G-bundle with curvature form Ω. To every \(\mathop{\mathrm{Ad}}\nolimits (G)\)-invariant polynomial on \(\mathfrak{g}\), one can associate the cohomology class [Λ] ∈ H (M) such that f(Ω) = π Λ. The cohomology class [Λ] is called the characteristic class of P associated to f. Denote by \(\mathop{\mathrm{Inv}}\nolimits (\mathfrak{g})\) the algebra of all \(\mathop{\mathrm{Ad}}\nolimits (G)\)-invariant polynomials on \(\mathfrak{g}\). The map

$$\displaystyle\begin{array}{rcl} w: \mathop{\mathrm{Inv}}\nolimits (\mathfrak{g})& \rightarrow & H^{{\ast}}(M) \\ f& \mapsto & [\varLambda ],\ \text{where }f(\varOmega ) =\pi ^{{\ast}}\varLambda,{}\end{array}$$
(32.3)

that maps each \(\mathop{\mathrm{Ad}}\nolimits (G)\)-invariant polynomial to its characteristic class is called the Chern–Weil homomorphism .

Example 32.3.

If the Lie group G is \(\mathop{\mathrm{GL}}\nolimits (r, \mathbb{C})\), then by Theorem B.10 the ring of \(\mathop{\mathrm{Ad}}\nolimits (G)\)-invariant polynomials on \(\mathfrak{g}\mathfrak{l}(r, \mathbb{C})\) is generated by the coefficients f k (X) of the characteristic polynomial

$$\displaystyle{\det (\lambda I + X) =\sum _{ k=0}^{r}f_{ k}(X)\lambda ^{r-k}.}$$

The characteristic classes associated to f 1(X), , f k (X) are the Chern classes of a principal \(\mathop{\mathrm{GL}}\nolimits (r, \mathbb{C})\)-bundle. These Chern classes generalize the Chern classes of the frame bundle \(\mathop{\mathrm{Fr}}\nolimits (E)\) of a complex vector bundle E of rank r.

Example 32.4.

If the Lie group G is \(\mathop{\mathrm{GL}}\nolimits (r, \mathbb{R})\), then by Theorem B.13 the ring of \(\mathop{\mathrm{Ad}}\nolimits (G)\)-invariant polynomials on \(\mathfrak{g}\mathfrak{l}(r, \mathbb{R})\) is also generated by the coefficients f k (X) of the characteristic polynomial

$$\displaystyle{\det (\lambda I + X) =\sum _{ k=0}^{r}f_{ k}(X)\lambda ^{r-k}.}$$

The characteristic classes associated to f 1(X), , f k (X) generalize the Pontrjagin classes of the frame bundle \(\mathop{\mathrm{Fr}}\nolimits (E)\) of a real vector bundle E of rank r. (For a real frame bundle the coefficients f k (Ω) vanish for k odd.)

Problems

32.1. Polynomials on a vector space

Let V be a vector space with bases e 1, , e n and u 1, , u n . Prove that if a function \(f: V \rightarrow \mathbb{R}\) is a polynomial of degree k with respect to the basis e 1, , e n , then it is a polynomial of degree k with respect to the basis u 1, , u n . Thus, the notion of a polynomial of degree k on a vector space V is independent of the choice of a basis.

32.2. Chern–Weil forms

In this problem we keep the notations of this section. Let e 1, , e n and u 1, … u n be two bases for the Lie algebra \(\mathfrak{g}\) with dual bases α 1, , α n and β 1, , β n, respectively. Suppose

$$\displaystyle{\varOmega =\sum \varOmega ^{i}e_{ i} =\sum \varPsi ^{j}u_{ j}}$$

and

$$\displaystyle{f =\sum a_{I}\alpha ^{i_{1} }\cdots \alpha ^{i_{k} } =\sum b_{I}\beta ^{i_{1} }\cdots \beta ^{i_{k} }.}$$

Prove that

$$\displaystyle{\sum a_{I}\varOmega ^{i_{1} } \wedge \cdots \wedge \varOmega ^{i_{k} } =\sum b_{I}\varPsi ^{i_{1} } \wedge \cdots \wedge \varPsi ^{i_{k} }.}$$

This shows that the definition of f(Ω) is independent of the choice of basis for \(\mathfrak{g}\).

32.3. Connection on P × I

Show that the 1-form \(\tilde{\omega }\) in (32.2) is a connection on P × I.

32.4. Chern–Weil homomorphism

Show that the map \(w: \mathop{\mathrm{Inv}}\nolimits (\mathfrak{g}) \rightarrow H^{{\ast}}(M)\) in (32.3) is an algebra homomorphism.